You are on page 1of 67

PRE-INTENSIVE EXAM NP1 anguish.

This human condition that confronts the


nurse gives rise to:
You as a nurse is expected to asses, contribute and *
preserve work environment that supports fulfilling 1 point
your ethical responsibility as a professional.
a. Unavoidable trust
1. Nurse Micah is assigned to the Emergency b. Ethical dilemma
Department. She is currently attending to a client c. Human indignation
with a complaint of acute abdominal pain. Which of d. Moral suffering
the following actions of Nurse Micah demonstrates
respect of client’s autonomy? 5. The nurse shows respect to human dignity when
* she observes which of the following situations
1 point when caring for the clients?
*
A. Complying when the physician attempts to 1 point
delegate obtaining informed consent
B. Describing the risks and benefits of the a. Asking the client’s priorities after assessing the
reasonable alternative to treatments client’s capabilities in the past and in the present
C. Facilitating and supporting client’s choices b. Evaluating response of client to the nursing care
regarding treatment options rendered by the health care team as planned
D. Notifying appropriate parties if a patient has not c. Constant monitoring of client’s condition and
given adequate information reporting any unusual occurrences to the health
team
2. The nurse obtain the blood pressure of a client d. Planning nursing care together with the client
and have a reading of 160/100mmHg. The nurse and immediate relative
did not tell the client the reading because she
believes that this information will upset him and SITUATION
consequently further elevate his blood pressure. Accuracy in the computation and administration of
This situation illustrates an example of: medications ordered is extremely important to
* avoid medication errors that may threaten the
1 point clients’ welfare.
A. Paternalism 6. A hypertensive client is ordered to receive 20
B. Self – determination mEq of Potassium Chloride. The bottle is labeled
C. Autonomy KCL elixir 10 mEq/ml. how many ml should be
D. Beneficence given?
*
3. Which of the following statements is true 1 point
regarding informed consent?
* a. 1.5 ml
1 point b. 2 ml
c. 0.5 ml
A. It is an ethical responsibility of nurses to provide d. 1 ml
client with opportunities to give informed consent
B. Nurses may not be legally liable if they know that 7. A client who is experiencing tachycardia is
informed consent was not obtained ordered to receive Digoxin 0.325 mg OD. The stock
C. It is ethical or legal for nurses to obtain informed is 0.25 mg/tab. How many tablet/s should be given
consent for procedures that are to be performed by to the client?
a physician *
D. It is not within a nurse’s domain of responsibility 1 point
to notify the health team if a client has not given an
informed consent for the procedure a. 1.5 tablets
b. 2 tablets
4. The nurse’s compassion is aroused when a c. 3 tablets
patient under her care is suffering and is in a d. ¾ tablet
prolonged life – sustaining machine. Many times
the nurse experiences feeling of uneasiness and 8. A client who have episodes of seizure is ordered
to received Dilantin 5mg/kg body weight is
ordered to a client who weighs 50lbs. the drug is to 4. A supportive and powerful coalition is developed
be administered in 3 equal doses. The label reads
Dilantin suspension 125mg/ml. how much 5. Improve communication and public relation
medication should be administered to the client? *
* 1 point
1 point
A. 3 and 4 only
a. 0.5ml B. 1 and 2 only
b. 1.0ml C. 2, 3 and 4 only
c. 1.5ml D. ALL of the options
d. 1.8ml
12. The team started the planning session with an
9. A post exploratory laparotomy client has an aim to revisit of the institution's vision-mission.
order for Meperidine Hydrochloride 50mg every 4 Vision pertains to:
hours PRN. The multiple vial dose vial is labeled *
50mg/ml. what is the correct dose to be 1 point
administered to this client when he complains of
pain? A. Value statement of the organization
* B. Preferred future of the organization
1 point C. Mission that anchors on certain specific task
D. Belief of the members
a. 0.5ml
b. 1.5ml 13. In the strength, weaknesses, opportunities and
c. 2.0ml threats (SWOT) analysis, the organizational
d. 1.0ml structure of the medical center including the chain
of command were analyzed. Which of the following
10. An order is given to a young adult to receive 1 describes an authoritative type of organization?
million units of penicillin. The stock on hand is *
Penicillin 500,000 units and the direction reads: 1 point
add 1.3ml to yield 2ml. What is the correct amount
to be administered? A. Authority and responsibility are delegated to the
* lowest level
1 point B. The organizational structure is flat
C. Communication comes from the different
a. 3ml directions
b. 2.5ml D. Control and communication flow is from top
c. 2ml down
d. 4ml
14. The team were unanimous in saying that the
SITUATION Medical Center is a people-oriented organization
: A Nurse manager of a medical center was tasked which promotes Empowerment. When there is a
to organize a core group that will conduct a decentralization of power it promotes the
strategic plan for their institution. The group following EXCEPT:
consists of key players from the different units of *
the health facility. 1 point
11. Which of the following are the benefits of A. Promotes long-range planning than short-term
coming up with a strategic plan? planning
B. Have voice in the governance of the institution
1. Enhance organizational capabilities of the C. Encourages creativity and commitment of the
constituents people
D. Increased morale of the personnel
2. Improve understanding among the members of
15. The outcome of the strategic plan session by the
the organization
representative groups is for the medical center to
have a shared governance. It means:
3. Realize the vision-mission of the institution
1. There will be increased authority and control D. Investigators work schedule
over their unit of assignments
19. You know that a good research problem should
exhibit the following characteristics. Which one is
2. Wider participation in decision-making before
NOT always included?
final decision is made
*
1 point
3. There will be autonomy and control of its own
practices A. Implies the feasibility of empirical testing
B. Indicates the hypothesis to be tested
4. Environment is democratic and participative C. Specifies the population being studied
* D. Clearly identified the variables/phenomenon
1 point under consideration

A. 1 and 2 only 20. You also know that a good research requires a
B. 2, 3, and 4 only thorough review of the literature. The first step in
C. 1 only searching for literature to locate all pertinent
D. ALL of the options sources is to
*
SITUATION 1 point
You are a member of research team of a health unit
that is tasked to conduct a study to present an A. Conduct print or computer search
evidence for the best nursing practice in your B. Identify key words and concepts to be searched
clinical setting. C. Ask help from the librarian about potential
references
16. A research idea or problem is generated D. Refer to the bibliographic lists of related studies
through the following, Except:
* SITUATION
1 point Spiritual care deals with the history, philosophy,
theories, principles, process, modes and
A. Critical review of literature interventions of spiritual care. Emphasis is made
B. Review of policy guidelines on the process of spiritual formation and the role of
C. Practical experience nurses in providing spiritual care.
D. Recommendation for funding source
21. The following statements are true in spiritual
17. Your team is interested to consider care nursing except
handwashing as a topic for research. The *
statement, "in what situations do staff nurses wash 1 point
their hands in the clinical areas?" This is an
example of a research ___________. a. Mobilizing the patient’s spiritual resources and
* patients’ expressed needs
1 point b. Developing a relationship of trust between the
nurse and the patient
A. Purpose c. Referral or utilize members of the team is not
B. Problem important for spiritual care as it is for other aspects
C. conceptual statement of care
D. hypothesis d. Awareness and respect of the patient’s culture,
social and spiritual preferences
18. In the final selection of a research problem,
aside from its significance to nursing practice, you 22. The following are vulnerable groups needing
will also have to consider the following criteria, spiritual care:
except:
* (1) Chronically ill patient (5) During
1 point Emergencies
A. Investigators interest to the problem
B. Researchability of the problem (2) Older adult (6) Patient with acute
C. Feasibility of addressing the research problem illness
(3) Dying and Bereavement (7) Children and (3) Provides support and care for those in the last
families phases of life- limiting illness

(4) During disasters (4) Recognizes dying as part of the normal process
* of living
1 point
(5) Affirms life and neither hastens nor postpones
A. 2, 4, 6
death
B. 1 only
C. 1, 3, 5, 7
D. All are correct (6) Focuses on quality of life for individuals and
their family caregivers
23. The following statements are true of Parish *
nursing also known as faith community nurse: 1 point
*
1 point a. 1, 3, 5
b. 2, 4, 6
a. A registered nurses with a minimum of 2 years c. All are true except 6
experience d. All are correct
b. There is conscious partnering of health issues
with the faith of the client and client’s family SITUATION
c. The core to this practice is in the intentional care Nurse Sophia is caring for Yannie, a 32 year old
of the spirit of those the PN assist primigravida at 39-40 weeks AOG was admitted to
d. All are correct the labor room due to hypogastric and lumbo-
sacral pains. IE revealed a fully dilated, fully effaced
24. Roles of a parish nurse includes all of the cervix.
following:
26. Andrea is immediately transferred to the DR
table. Which of the following conditions signify that
(1) Health advisor
delivery is near?
(2) Educator on health issues
1. A desire to defecate
(3) Advocate/ resource person
2. Begins to bear down with uterine contraction
(4) Liason to faith and community resources
3. Perineum bulges
(5) Teacher and volunteers and developer of
4. Uterine contraction occur 2-3 minutes intervals
support groups
at 50 seconds duration
*
(6) Healer of body, mind, spirit and community 1 point
*
1 point A. 1,2,3
B. 1,2,3,4
A. 1, 3, 5 C. 1,3,4
B. b. 2, 4, 6 D. 2,3,4
C. All are true except 6
D. All are correct 27. Artificial rupture of the membrane is done.
Which of the following nursing diagnoses is the
25. The following statement are true of palliative/ priority?
hospice care *
1 point
(1) Excellent, evidence based medical treatment
a. High risk for infection related to membrane
(2) Vigorous care of pain and symptoms rupture
throughout illness b. Potential for injury related to prolapse cord
c. Alteration in comfort related to increasing when you are ready to give the injection. Your best
strength of uterine contraction action would be to:
d. Anxiety related to unfamiliar procedure *
1 point
28. Yannie complains of severe abdominal pain and
back pain during contraction. Which two of the a. Inject it in the playroom; insulin injections do not
following measures will be MOST effective in hurt.
reducing pain? b. Tell her to come outside the playroom for the
injection.
1. Rubbing the back c. Ask the other children if they would mind if you
gave the injection in the playroom.
2. Effleurage d. Ask the girl if she would mind if you gave the
injection in the playroom.
3. Imagery 32. Diabetes insipidus (DI) is suspected in a child.
His parents asked Nurse Mak how often the child
4. Breathing techniques needs insulin injections in a day. Nurse Mak
* answers correctly by stating:
1 point *
1 point
A. 2,4
B. 2,3 a. “The child would need an intermediate- and long-
C. 1,4 acting insulin given 2 hours and before night time
D. 1,2 respectively.”
b. “You seem very anxious. Do you want to talk
29. Lumbar epidural anesthesia is administered. about your fears?”
Which of the following nursing responsibilities c. “Your child’s condition does not necessarily need
should be done immediately following procedure? insulin regimens.”
* d. “I will ask your doctor first.”
1 point
33. A patient with Cushing’s syndrome due to a
osition from side to side primary tumor is expected to have elevated levels
b. Administer oxygen of the following, except:
c. Increase IV fluid as indicated *
d. Assess for maternal hypotension 1 point
30. The nurse is caring for a woman in labor. The a. Adrenocorticotropic hormone
woman is irritable, complains of nausea and vomits b. Antidiuretic hormone
and has heavier show. The membranes rupture. c. Cortisol
The nurse understands that this indicates: d. None is an exception
*
1 point 34. Isabella is suspected to have diabetes mellitus
(DM). Her first test reveals elevated blood sugar.
a. The woman is in transition stage of labor She was required to undergo a second test of her
b. The woman is having a complication and the blood sugar. Which of these results would confirm
doctor should be notified presence of DM?
c. Labor is slowing down and the woman may need *
oxytocin 1 point
d. The woman is emotionally distraught and needs
assistance in dealing with labor. a. Fasting blood sugar 104 mg/dL
b. Random blood sugar 190 mg/dL
SITUATION c. Fasting blood sugar 130 mg/dL
Nurse Mak is caring for pediatric clients who have d. Random blood sugar 122 mg/dL
different metabolic and endocrine disorders.
35. Which toy would you expect to provide the best
31. A school-age child needs 5 units of regular therapeutic play for a child who has to receive daily
insulin to be administered. She is in the playroom medicine injections?
* 38.Nurse Regine is handling another patient with
1 point myelomeningocele, a type of spina bifida. Nurse
Regine knows that neural tube disorders like spina
a. Anatomically-correct puppet bifida is prevented by prenatal intake of which of
b. Doll with a cast in place. the following?
c. Syringe to practice injections *
d. Stuffed bear with Band-Aids. 1 point
SITUATION a. Vitamin B9
There are various pediatric disorders that require b. Vitamin B6
comprehensive assessment and nursing c. Vitamin B12
interventions. The following scenarios refer to d. Vitamin B1
health problems of children.
39. The patient with myelomeningocele is placed
36. Ian is a pediatric patient of Nurse Regine. Ian is under a warmer without any clothing. How should
diagnosed with Duchenne’s Muscular Dystrophy Nurse Regine care for the patient’s sac?
(DMD). Because of this condition, when Ian tries to *
walk, he is said to “walk up his front” – pressing his 1 point
hands against his ankles and thighs. Regine notes
sign as: a. Leave it uncovered.
* b. Frequently rub petroleum jelly on the sac
1 point covering.
c. Apply sterile, moist, nonadherent dressing over
a. Gower’s sign the sac.
b. Chadwick’s sign d. Cover with sterile gauze moistened with 40%
c. Ortolani’s sign ethyl alcohol.
d. Barlow’s sign
40. In feeding the infant with the
37. To maintain the optimal functioning of Ian’s myelomeningocele sac, how should Nurse Regine
muscles, select the interventions that Nurse Regine position him to prevent complications?
should include in her care plan and discharge plan. *
1 point
i. Maintain complete bed rest as long as
possible. a. Place the infant in a supine position with two
pillows under the head
ii. Perform stretching exercises, strength b. Keep the patient in prone position with the head
and muscle training as much as the client can turned to the side.
tolerate. c. Place the infant supine with pillows on one side
of his back.
d. Request for a nasogastric tube to be inserted.
iii. Perform breathing exercises to increase
and maintain lung vital capacity. SITUATION
You are part of the research department in the
iv. Comply to follow-up in physical therapy Department of Health. As a nurse researcher, you
have a good understanding on the different
v. Encourage influenza and pneumococcal concepts and terms utilized in carrying out the
vaccines research process.
41. In one of the studies conducted, the hypothesis
vi. Encourage parents to perform all Jim’s formulated was stated, “Baccalaureate degree-
ADLs to avoid fatigue. prepared nurses will practice more palliative
* nursing measures on a client in an ICU than will
1 point associate degree-prepared nurses.” The
independent variable here is:
a. i, iii, iv
*
b. i, ii, iii, iv, v
1 point
c. ii, iii, iv
d. ii, iii, iv, v
a. Baccalaureate degree-prepared nurses 46.Which of the following food products should be
b. Associate degree-prepared nurses eliminated in the diet of a child with
c. Palliative nursing measures Phenylketonuria?
d. Type of educational background of nurses *
1 point
42. In a certain study, the researcher was asked to
include the subjects’ gender. Gender is classified in a. bread
what scale of measurement? b. pineapple
* c. milk
1 point d. potato
a. Ordinal 47. In clients with Phenylketonuria, at which
b. Ratio developmental period would the dietary
c. Nominal restrictions be easily applied?
d. Internal *
1 point
43. Empiricism refers to:
* a. Infancy
1 point b. Toddlerhood
c. Preschool years
a. Making generalizations from a specific d. School years
observation
b. Gathering evidence rooted in objective reality 48. Which of the following is recommended for
c. Verifying assumptions on which the study was newborns and infants who are diagnosed with
based Galactosemia?
d. Deciding specific productions from *
generalizations 1 point
44. In conducting a research, you did not collect any a. breastmilk
information that would link to the identification of b. commercial milk formula
the participants to their responses. The ethical c. fruit juices
guideline observed is: d. soy-based formula
*
1 point 49. For newborns diagnosed with Maple Syrup
Urine Disease, which of the following vitamins
a. Privacy of participants should be increased in their diet?
b. Autonomy *
c. Confidentiality of information 1 point
d. Anonymity of respondents
a. Thiamine
45. As you write your introduction for your study, b. Pyridoxine
you must take all of the following considerations, c. Tochopherol
except: d. Vitamin K
*
1 point 50.All of the following can trigger hemolysis in
children with Glucose-6-Phoshate-Dehydrogenase
a. Presenting every detail of the study concisely Deficiency, except:
b. Omitting the major findings of the study *
c. Careful selection of words to avoid redundancy 1 point
d. Using a deductive presentation of the study
a. fava beans
SITUATION b. naphthalene balls
Aside from knowing the diseases which are c. Aspirin
currently included in the Philippines’ Newborn d. Junk foods
Screening Program, nurses must also be equipped
with the knowledge on how to properly care for SITUATION
newborns who tested positive for any of the inborn Nurse Sunshine works in the orthopedic unit of the
errors of metabolism. pediatric ward.
51. A 13 year old female with structural scoliosis c. A primary lateral curvature with a compensatory
has Harrington rods inserted. Nurse Sunshine second curve
knows that the best position during the post- d. A compensatory mechanism in children who
operative period is: have unequal leg lengths and refractive errors
*
1 point SITUATION
A major continuing and non-negotiable task of
a. High fowler’s every nurse in the care of infants and children at
b. Semi-fowler varying stages of their growth and development is
c. supine in bed the application of her assessment skills. The
d. side-lying following questions apply.
52. Which of the following observations does Nurse 56. Nurse Ivee inquires about the activity level of a
Sunshine expect to see in her patient with thoracic 3-year-old under her care. The mother states that
scoliosis? the child loves to play at the park, and the nurse
* encourages the mother to continue physical
1 point activities. What important principle guides the
nurse’s response?
a. The patient walks with a waddling gait *
b. The patient’s sternum is protruding 1 point
c. The patient’s lower legs are edematous
d. The patient’s thoracic area is asymmetrical a. allowing the toddler to walk, run and hop
enhances the child’s kinesthesia
53. A child with bilateral clubfeet is going home b. socialization with other toddlers helps develop
with corrective braces after having both casts communication skills
removed. Nurse Sunshine is completing discharge c. maternal bonding is enhanced through play
teaching for the parents. Which of the following is d. only emotionally happy child can enjoy the park
the highest priority?
* 57. The father of a 2½ year old asked nurse Ivee
1 point how to prevent early childhood dental cavities. The
best response by nurse Ivee would be:
a. Keep the braces concealed under long pants *
b. Remove the braces whenever the child is in 1 point
public
c. Use lotion and then powder to prepare the skin a. “Let the child watch you brush your teeth so that
before applying the corrective braces to the legs he can learn how to do it himself.”
d. Increase the time of interval of wearing the b. “Your child has only baby teeth; these will
braces and have the child wear them eventually as eventually fall out and so there is no need to
much as possible worry.”
c. “Take the child to the dentist to see if he has any
54. Which of the following positions of the femur is cavities.”
accurate in relation to the acetabulum in a child d. “Make sure your child’s diet is nutritious, and
with congenital hip dislocation? limit snacks high in sugar.”
*
1 point 58. In caring for a 3-year-old, Nurse Ivee knows
that she needs to obtain the height of the child as
a. Anterior part of routine health screening. To obtain an
b. Inferior accurate measurement, the child must:
c. posterior *
d. superior 1 point
55. Nurse Sunshine knows that structural scoliosis a. remove his shoes and stand upright, with head
is not: level
* b. stand with his feet wide apart
1 point c. be measured in a recumbent position
a. Idiopathic d. face the wall as he is measured
b. Accompanied by damage to the vertebrae
59. The mother of a 3-year old child also under a. Provide emotional support to the parents
Nurse Ivee’s care tells her that the child has b. Monitor signs of increased intracranial pressure
frequent nightmares. The statement by the mother c. Monitor vital signs
that indicates the need for more teaching is: d. promote normal growth and development of the
* child
1 point
64. At nine months old, which of the following
a. “I read her a story until she calms down.” behaviors is indicative that his development is
b. “I stay with her a little while to reassure her.” delayed. He can:
c. “I usually talk quietly and rub her back to *
reassure her.” 1 point
d. “I take her to my bed so she will calm down.”
a. Sit with support
60. Our school curricula now include educating the b. Pull himself up to his feet with assistance
young regarding human sexuality. What is the most c. Swallow liquid from a cup
appropriate age group for the nurse to incorporate d. Handle semi-solid food
these in her instructions?
* 65. Kenneth underwent ventriculo-peritoneal
1 point shunting. What is the best position for Kenneth
post-operatively?
a. 9 years old *
b. 15 years old 1 point
c. 13 years old
d. 11 years old a. Flat on bed
b. Trendelenburg
SITUATION c. head is elevated
Kenneth James, a 9 month old infant is admitted in d. side lying
the pediatric unit due to enlarged head
circumference, bulging fontanelle and sunset eyes. SITUATION
He is diagnosed to have hydrocephalus Safety and quality during the birth – giving episode
is the nurse’s principal concern. The following
61. Which of the following is not true about questions apply.
hydrocephalus. It is
* 66. The nurse is preparing a woman for epidural
1 point anesthesia. The woman asks “why is my IV running
so fast? It feels so cold” what is the appropriate
a. A disorder that occurs only at birth reply of the nurse?
b. A problem of over production of CSF *
c. An obstruction of flow of CSF in the brain’s 1 point
ventricular circulation
d. An under absorption of CSF a. “IV hydration helps prevent the blood pressure
from dropping so low”
62. Magnetic resonance imaging is done. Which of b. “Don’t worry. This is a routine procedure in
the following results does not confirm the preparation for an epidural anesthesia”
diagnosis of hydrocephalus? c. “I’ll slow the IV down so you won’t feel so cold”
* d. “IV fluids help prevent spinal headaches”
1 point
67. A woman arrived to the labor suite and states “I
a. Site of CSF blockage have water leaking down my legs” What
b. Enlarged cranium assessment is most appropriate in this situation?
c. atrophied brain *
d. enlarged ventricles 1 point
63. The top priority in rendering nursing care to a. urine test for protein
this patient is: b. DTRs
* c. Fern test
1 point d. Blood pressure check
68. it is most likely that the physician would 1. Encourage, protect and support the practice of
consider performing an amnioinfusion when breastfeeding
external fetal monitor tracing shows:
* 2. Save on costs for utilities and personnel for a
1 point newborn nursery
a. Flat line without variability and no decelerations
b. Occasional mild variable decelerations and 3. Create an environment where basic physical,
moderate variability present emotional, and psychological needs of mothers and
c. Deep variable decelerations with every infants are fulfilled
contraction
d. Consistent early decelerations, variability 4. Teach the mother to take responsibility for
present and occasional accelerations caring for her newborn right after her delivery
*
69. A woman carrying a breeched fetus is 1 point
scheduled for external version. She says to the
nurse “I’m really scared of the procedure, will it a. 2 and 3 are correct
hurt badly?” What is the best reply of the nurse? b. 2 and 4 are correct
* c. 1 and 2 are correct
1 point d. 1 and 3 are correct

a. “The procedure can be quite uncomfortable, but 72. Soon after both mother and baby were settled
it is best for your baby. You want to do what is best in their hospital room, the pediatrician, who is now
for the baby, right?” in the hospital, came to see the baby. After
b. “Sometimes the procedure is uncomfortable. If it reviewing the baby’s condition immediately after
becomes too painful, let the doctor know and she delivery, she asks, “Is there a standard milk formula
will stop the procedure. ” the hospital prescribes or would want us to use?”
c. “You can do it. I’ll hold your hand throughout the Nurse Melody was quick to respond, “I am sorry,
procedure and you should be just fine.” but shouldn’t we be adhering to the National Milk
d. “Don’t worry. An external version procedure is Code Act of 2006 and the Expanded Breastfeeding
not painful.” Act of 2009?” This reflects the nurse’s role as:
*
70. Which statement by a postpartum woman after 1 point
caesarean delivery indicates that further discharge
teaching is needed? A. strong supporter of the Bureau of Food and Drug
* (BFAD) in law enforcement
1 point B. patient-advocate and law-abiding practicing
health worker
a. “I need to hold my incision when I cough” C. representative of the Professional Regulation
b “My mother will come to help me when I get Commission (PRC) enforcing standards
home.” D. ordinary citizen exercising police powers
c. Being tired may increase the pain I feel.”
d. “The incision needs to be covered with a 73. Socorro is very much aware that in the case of
bandage” the newborn, within the first hour after birth, that
which is necessary?
SITUATION *
Socorro, a nurse working in the OB ward of a 1 point
second level hospital received a post normal
delivery mother back in the ward with her healthy A. Immediate initiation of breastfeeding with
newborn baby girl. Rooming- In is newly practiced complementary milk formula.
in this hospital B. Immediate initiation of breastfeeding continued
within 6 months up to 2 years and beyond without
71. Socorro is aware that in accordance with R.A. complementary milk formula. Complementary
7600 of 1992, the purpose of the “rooming- in” feeding started only at 6 months.
national policy are two-fold; C. Actual initiation of breastfeeding, continued with
complementary milk formula within 6 months up
to 2 years and beyond.
D. Initiation of breastfeeding but when not SITUATION
possible, immediate introduction of The nurse is caring for a G3P1 postpartum client
complementary formula feeding. who just had her delivery.
74. After her duty hours, a milk company 76. The nurse is checking for the patients VS and
representative approached Socorro offering her noticed that her temperature is increased. The
part time job with them. Socorro turned down the mother noticed that she also feels really warm and
offer because this is in conflict with her desire to thirsty. What would the nurse tell the patient?
promote breastfeeding. In addition, Section 32 of *
Administrative Order 2006-0012 as implementing 1 point
rules and regulations to Executive Order No. 51
(Milk Code Policy): a. Tell the patient that what she is experiencing is
* normal within 24 hours.
1 point b. Tell the patient that what she is experiencing is
normal within 48 hours.
A. provides that it is generally a choice for the c. Tell the patient that she has a starting infection.
health professional. d. Tell the patient that she has a good immune
B. provides that it is the primary responsibility of system.
the health workers to promote, protect, and
support breastfeeding and appropriate infant and 77. After 3 days, the patient had a urinalysis. The
young child feeding and that no assistance, nurse noticed that the patient’s BUN is relatively
logistics, or training from milk company is high. What would the nurse do?
permitted. *
C. allows health workers to do public 1 point
endorsements for as long as it is outside of their a. Refer to the attending physician.
duty. But accepting the offer would create conflict b. Note the occurrence as normal.
in professional interest which will rebound to c. Assess the patient’s level of consciousness.
unethical conduct. d. Confer with the other nurses regarding the
D. is actually silent about the matter. laboratory result.
75. Socorro attended a meeting in the hospital 78. Knowing that the patient has high levels of BUN
representing her ward, where she pushed for in her urine. What would the nurse tell the client
adherence to the “Expanded Breastfeeding about her diet?
Promotion Act of 2009” RA 7600 (Rooming-In and *
Breastfeeding Act). This measure provides the 3rd 1 point
vital component of the breastfeeding law
(PROVISION OF SUPPORT for the practice of a. Tell the client to decrease her intake of CHON-
breastfeeding) by: rich food.
* b. Tell the client to increase her intake of CHON-rich
1 point food.
c. Tell the client to increase the intake of CHO-rich
A. updating the penalty provisions of the former food.
Aquino EO No. 51 and AO No. 2006-0012. d. Tell the client to decrease her intake of CHO-rich
B. maintaining the provision of facilities for food.
breastmilk collection and storage (e.g. milk banks)
and establishment of “lactation stations” with 79. The patient is about to be discharged from the
adequate support facilities, and providing clinic. The patient asks the nurse of when her
incentives and sanctions thereto. menses will return if she will not continuously
C. mandating the provision of facilities for breastfeed her baby. What will be the proper health
breastmilk collection and storage (e.g. milk banks) teaching of the nurse?
and establishment of “lactation stations” with *
adequate support facilities, and the integration of 1 point
breastfeeding education in the curricula.
D. integrating the key provisions of EO 51 and AO a. The patient will have her menstrual flow back on
No. 2006-0012 track in 6-10 weeks after giving birth.
b. The patient will have her menses 3-4 months severe cramps. Vaginal examinations reveals that
after birth. her cervix is dilated and tissue is
c. The patient will have varying menstrual flow. bulging through the os. Which of the
d. The client will experience some pain in the following would be the best procedure in
abdomen while breastfeeding her baby. managing this new condition?
*
80. The client after she was discharged came back 1 point
to the clinic in her 3rd postpartum day. She noticed
that her breasts are really full and that she feels A. dilation and currettage
that it’s really tense. What will the nurse tell the B. oxytocin stimulation
patient? C. saline infusion
* D. Shirodkar procedure
1 point
84. Which of the following positions should the
a. Tell the client that it is a sign of mastitis. nurse place the patient who has been diagnosed
b. Tell the client that she is experiencing an with a prolapsed cord
overproduction of milk. *
c. Tell the client that the breasts become tender and 1 point
full at the start of the 3rd postpartum day.
d. Tell the patient that she’s just too excited about a. supine
what’s happening to her. b. left lateral recumbent
c. Trendelenberg
SITUATION d. high fowlers
Marie Amoure calls her pre-natal clinic to
report that, she had intermittent lower 85. A woman has been in preterm labor off and on
abdominal cramping and occasional spotting for for 2 weeks. She is 28 weeks pregnant. It seems
the last 24 hrs. Her last menstrual period was eight inevitable that she will delivery soon. Which of the
weeks ago. Two weeks ago she had a positive following medications would the nurse prepare to
pregnancy test. give this patient?
*
81. The most likely diagnosis for Dollies condition 1 point
on the basis of the information presented is:
* a. celestone
1 point b. magnesium sulfate
c. pitocin
a. inevitable abortion d. Terbutaline
b. incomplete abortion
c. threatened abortion SITUATION
d. spontaneous abortion As a newly registered nurse, Nurse Nenita was
employed in a government hospital and is assigned
82. Marie Amoure comes to clinic. Pelvic in the maternity unit. Here, she takes care of
examination, which reveals vaginal bleeding but no gravidomorbid clients. It is essential to know the
dilatation of the cervix, confirms the normal and abnormal changes in pregnancy in
tentative diagnosis. Which of the following should order to deliver the best nursing care. The
not be considered in the management of Kate’s following questions apply.
disorder?
* 86. During the night shift, a woman is hospitalized
1 point for the treatment of severe preeclampsia. Which of
the following represents an unusual finding for this
A. bed rest condition?
B. abstinence from intercourse *
C. Administration of mild sedative 1 point
D. Administering of diethylstilbestrol (Synthetic
estrogen) A. Proteinuria 4+.
B. Blood pressure 160/100.
83. A week later Marie Amoure comes back to the C. Generalized edema.
emergency room because of heavy bleeding and D. Convulsions.
87. As a nurse in the maternity unit, Janine knows clinic. The following clients consulted Nurse
that the action of hormones during pregnancy Rancelle, thus, she performed health teachings.
affect the body by?
* 91. During a visit to the prenatal clinic, Mia is at 32
1 point weeks’ gestation and complains of heartburn. After
Rancelle’s health teaching, the client needs further
A. Raising resistance to insulin. instruction when Jenny says she must do what?
B. Blocking the release of insulin from the pancreas. *
C. Preventing the liver from metabolizing glycogen. 1 point
D. Enhancing the conversion of food to glucose.
A. Avoid highly seasoned foods.
88. A 28-year-old woman has had diabetes mellitus B. Avoid laying down right after eating.
since she was an adolescent. She is 8 weeks C. Eat small, frequent meals.
pregnant and is admitted under the care of Nurse D. Consume liquids only between meals.
Nenita. Hyperglycemia during her first trimester
will have what effect on the fetus? 92. Rancelle is teaching a new prenatal client about
* her iron deficiency anemia during pregnancy.
1 point Which statement indicates that the client needs
further instruction about her anemia?
A. Hyperinsulinemia. *
B. Excessive fetal size. 1 point
C. Malformed organs.
D. Abnormal positioning. A. “I will need to take iron supplements now.”
B. “I may have anemia because my family is of Asian
89. Nenita is caring for a young diabetic woman descent.”
who is in her first trimester of pregnancy. As the C. “I am considered anemic if my hemoglobin is
pregnancy continues the nurse should anticipate below 11 g/dL.”
which change in her medication needs? D. “The workload on my heart is increased when
* there is not enough oxygen in my system.”
1 point
93. An antenatal primagravid client has just been
A. A decrease in the need for short-acting insulins. informed that she is carrying twins. The plan of
B. A steady increase in insulin requirements. care includes educating the client concerning
C. Oral hypoglycemic drugs will be given several factors that put her at risk for problems during the
times daily. pregnancy. Nurse Rancelle realizes the client needs
D. The variable pattern of insulin absorption further instruction when she indicates carrying
throughout the pregnancy requires constant close twins puts her at risk for which of the following?
adjustment. *
1 point
90. Methylergonovine (Methergine) is prescribed
for a client with postpartum hemorrhage. Before A. Preterm labor.
Nenita administers the medication, she contacts B. Twin-to-twin transfusion.
the health care provider who prescribed the C. Anemia.
medication if which condition is documented in the D. Group B Streptococcus.
client’s medical history?
* 94. A 30-year-old multigravid client has missed
1 point three periods and now visits the prenatal clinic
because she assumes she is pregnant. She is
A. Hypotension experiencing enlargement of her abdomen, a
B. Hypothyroidism positive pregnancy test, and changes in the
C. Diabetes mellitus pigmentation on her face and abdomen. These
D. Peripheral vascular disease assessment findings reflect this woman is
experiencing a cluster of which signs of pregnancy?
SITUATION *
Nurse Rancelle is addressing concerns of several 1 point
clients in the out-patient department of a lying-in
A. Positive.
B. Probable. trace of protein. The nurse should advise the client
C. Presumptive. that which of the following factors increases her
D. Diagnostic. risk for preeclampsia?
*
95. An antenatal client receives education 1 point
concerning medications that are safe to use during
pregnancy. Rancelle evaluates the client’s A. Total weight gain.
understanding of the instructions and determines B. Short stature.
that she needs further information when she states C. Adolescent age group.
which of the following? D. Proteinuria.
*
1 point 99. After instructing Rhufa, who is now at 38
weeks’ gestation, about how preeclampsia can
A. “If I am constipated, Milk of Magnesia is okay but affect the client and the growing fetus, nurse
mineral oil is not.” Coachie realizes that the client needs additional
B. “If I have heartburn, it is safe to use Tums, instruction when she says that preeclampsia can
Rolaids, Mylanta, and Maalox.” lead to which of the following?
C. “I can take Tylenol if I have a headache.” *
D. “If I need to have a bowel movement, Ex-Lax is 1 point
preferred.”
A. Hydrocephalic infant.
SITUATION B. Abruptio placentae.
Nurse Coachie is a community health nurse and has C. Intrauterine growth retardation.
paid particular attention to Rhufa, a 16 year old D. Poor placental perfusion.
client at 30 weeks gestation.
100. Pat was rushed to the hospital and is
96. Nurse Coachie is developing a teaching plan for experiencing pain during the first stage of labor.
a client entering the third trimester of her The nurse on duty should instruct her to do all of
pregnancy. He should include all of the following in the following, except:
the plan, except: *
* 1 point
1 point
A. Walk in the hospital room.
A. Ambivalence concerning pregnancy. B. Use slow chest breathing.
B. Experimenting with mothering roles. C. Lightly massage her abdomen.
C. Realignment of roles and tasks. D. Sip ice water.
D. Trying various caregiver roles.
97. Rhufa, who is being monitored at home with
home nursing visits, is diagnosed with mild
preeclampsia and has gained 2 lb in the past week.
Her current blood pressure is 144/92 mm Hg.
Which assessment finding would require further
action by the Nurse Coachie?
*

1 point
A. Occasional headache.
B. Frequent voiding in large amounts.
C. 1 + pedal edema.
D. 3 + protein on urine dipstick.
98. At 32 weeks’ gestation, Rhufa, a 15y/o
primigravid client who is 5 feet, 2 inches tall has
gained a total of 20 lb, with a 1-lb gain in the last 2
weeks. Urinalysis reveals negative glucose and a
PRE-INTENSIVE EXAM NP2 C. Municipal Health Officer
D. Any of these health professionals
Local Government Code known as R.A. 7160
brought major shift in the roles of the Department 5. Community health nursing is a developmental
of Health. Under this law, all structures, personnel service. Which of the following best illustrates this
and budgetary allocations from the provincial statement?
health level down to the barangays were devolved *
to the local government units (LGUs) to facilitate 1 point
health service delivery
A. Community health nursing is intended primarily
1. R.A. 7160 mandates devolution of basic services for health promotion and prevention and
from the national government to local government treatment of disease.
units. Which of the following is the major goal of B. The goal of community health nursing is to
devolution? provide nursing services to people in their own
* places of residence.
1 point C. The community health nurse continuously
develops himself personally and professionally.
A. To make basic services more accessible to the D. Health education and community organizing are
people necessary in providing community health services.
B. To strengthen local government units
C. To allow greater autonomy to local government SITUATION
units You are now in the entry phase of community
D. To empower the people and promote their self- organizing process, after establishing rapport with
reliance the people in Barangay San Miguel. The following
questions apply.
2. Lavinia recently passed the board exam and now
has her license to practice. She wants to become a 6. With the community health activities started and
Public Health Nurse. Where will she apply? in the entry phase, which of these activities should
* you not include?
1 point *
1 point
A. Department of Health
B. Provincial Health Office a. Information campaign on health programs
C. Regional Health Office b. Project Management
D. Rural Health Unit c. Core group formation
d. Conduct of deepening social investigation
3. The public health nurse is the supervisor of rural
health midwives. Which of the following is a 7. Identification of potential leaders is crucial
supervisory function of the public health nurse? during the entry phase. Which of these
* characteristics may not be necessary to an efficient
1 point and effective community leader?
*
A. Referring cases or patients to the midwife 1 point
B. Providing nursing care to cases referred by the
midwife a. Responsive and willing to work for change
C. Formulating and implementing training b. Must have relatively good communication skills
programs for midwives c. A respected member of the community
D. Providing technical guidance to the midwife d. A college graduate with management skills
4. One of the participants in a hilot training class 8. The best technique in identifying potential
asked you to whom she should refer a patient in leaders in the community proven to be affective is
labor who develops a complication. You will to:
answer would be, to the: *
* 1 point
1 point
a. Ask community residents to directly name
A. Public Health Nurse person whom they consider as community leaders
B. Rural Health Midwife
b. Ask volunteers who are willing to become 12. Community organizing is an important part of
community leaders the community nursing function. Given the
c. Review family background properties and following elements: choosing an organizational
academic records of community residents structure, identifying and recruiting members,
d. Observe people who are active in small defining mission, vision and goals, clarifying roles
mobilization activities that motivate residents to and responsibilities at which stage do these
start working elements belong?
*
9. The core group is composed of individuals in the 1 point
community who possess leadership potentials
organized into cohesive working unit. The core A. Program maintenance-consolidation
group works with the team in: B. Community analysis/diagnosis
* C. Design and initiation
1 point D. Dissemination-reassessment
a. Selecting community activities according to their 13. At which phase of the community
preferences organizing process are the leaders or groups given
b. Monitoring the performance of the barangays training to develop their knowledge, skills and
officials attitude in managing their own programs/
c. Setting up community organization that will *
serve the interest of a sector in the community 1 point
d. Mobilizing the community to act on their most
immediate felt needs and participate in the a. Sustenance and strengthening phase
delivery of essential health b. Pre- entry phase
c. Organizing- building phase
10. The nurse as a community health practitioner d. Entry phase
functions as a community:
* 14. When you have already organized, follow up
1 point home visits are likewise needed. Which of the
following should you do first?
A. Facilitator *
B. Planner 1 point
c. Leader
d. Evaluator A. Wash your hands and perform the necessary
procedure
SITUATION B. Explain the purpose of the visit
You are a beginning community health nurse in C. Greet the patient/ resident, introduce self
your Health Center. You are aware that meaningful D. Do an environmental surveillance
integration is essential for you to carry out your
roles and functions. The following questions apply. 15. Which of the following priority conditions will
be determining factors on the frequency of home
11. Integration is the process of establishing health visits?
rapport with the people in the community. It can *
best be achieved by: 1 point
*
1 point A. The need for health teachings and level of family
understanding
A. Bringing in some gifts to win acceptance of the B. Acceptance of the family of their health concerns
people in the community and their coping strategies
B. Cleaning the house and doing other household C. The results of the studies conducted and level of
chores family understanding
C. Conversing with the people where they are D. Administration of medications and the schedules
participating in social activities set
D. Sponsoring a sports festival to have an
opportunity to meet more people SITUATION
A relevant DOH program addresses community
health conditions particularly Tuberculosis. Again,
safety and quality care is every nurse’s concern. SITUATION
The following questions apply. The case of Acquired Immunodeficiency Syndrome
(AIDS) in the Philippines has been rapidly
16. Directly Observed Treatment Short-course increasing. Official records of the Department of
(DOTS) can do all of the following except: Health (DOH) showed that the number of HIV cases
* rose to 709 in 2009, compared to the 528 recorded
1 point the previous year. Nurses have a pivotal role in
A. stop resistance to anti-TB drugs educating the public to prevent an increase in
B. cure TB patients occurrence of this transmissible disease.
C. increase health care cost 21. The nurse looks for results of which laboratory
D. prevent new infection among children and measurement that provides a reliable indicator of
adults lymphocyte status in a client with HIV infection?
17. Which vital statistics relating to Tuberculosis *
(TB) in our country is inaccurate? 1 point
* a. B lymphocytes
1 point b. T-cytotoxic cells
A. TB is the 6th leading cause of illness among c. T-helper cells (CD4)
Filipinos d. Natural killer cells (NK)
B. TB is the 6th leading cause of deaths among 22. A male client who has acquired
Filipinos immunodeficiency syndrome (AIDS) asks why oral
C. The Philippines is among the 22 highly burdened progesterone (Megace) is being prescribed for
poor countries in the world treatment. What is the nurse’s best response?
D. Most TB patients belong to the 0-15 age groups *
18. The elements of TB DOTS that need to be 1 point
fulfilled include all of the following except: a. “Megace is used to treat the nausea associated
* with this infection.”
1 point b. “Megace is used as an appetite stimulant to boost
A. regular chest x-ray for diagnosis nutritional support.”
B. standardized recording and reporting of TB data c. “Megace is used as an antineoplastic agent for
C. regular supply of anti-TB drugs palliative treatment.”
D. supervised treatment by a treatment partner d. “Megace provides symptomatic relief of
constipation.”
19. Pulmonary TB is suspected when client
manifests which symptoms? 23. Which one of the following suggestions by the
* nurse would be most helpful to a human
1 point immunodeficiency virus (HIV) positive client who
has altered taste perception?
A. hematemesis *
B. sudden loss of weight 1 point
C. cough for more than 2 weeks
D. precordial chest pain a. Drink plenty of salty broths and other fluids to
stimulate taste buds.
20. Which of the following statements about TB b. Try zinc supplementation to improve taste
treatment is incorrect? perception.
* c. Increase intake of meat to at least one serving per
1 point day.
d. Avoid using plastic eating utensils.
A. combination of 3-4 anti-TB drugs is the
treatment of choice 24. Which of the following suggestions would the
B. single drug is appropriate nurse give to a client with human
C. treatment renders patients non-infectious and immunodeficiency virus infection to best alleviate
cured nausea?
D. tuberculosis is a curable disease *
1 point 5. Slowly inject sterile irrigant into the catheter
and bladder
a. Drink liquids with meals.
b. Eat high-fat foods.
6. Remove the clamp and allow irrigant to drain
c. Eat small, frequent meals.
into the collion bottle/bag
d. Lie down after eating.
*
25. The nurse is caring for a pediatric client with 1 point
acquired immunodeficiency syndrome (AIDS).
a. 1,2,3,4,5,6
Which activity by the nurse should be reported to
b. 2,1,3,5,4,6
the employee health department as an exposure for
c. 2,3,1,4,5,6
the nurse?
d. 3,2,1,4,5,6
*
1 point 28. When a client has a retention catheter, the
nurse is expected to:
a. While flushing out the used bedpan, fluid
*
splashes in the nurse’s eyes.
1 point
b. The nurse is stabbed with a sterile syringe to be
used to draw up the client’s medications. a. Clean the urinary meatus and the adjacent skin
c. Nurse does not wear a mask while in the client’s periodically
room. b. Flush the catheter as needed
d. During the bath, the nurse removes gloves when c. Perform perineal flushing as needed
giving a backrub on intact skin. d. Encourage liberal amount of fluid intake
SITUATION 29. After removing the indwelling catheter the
Urethral catheterization requires a physician’s client complains of difficulty in her first attempt to
order. Special care and strict aseptic technique urinate. The nurse explains that this is due to:
must be observed for clients with indwelling *
catheter. 1 point
26. A day after the insertion of the urinary a. Irritation of the urethra
retention catheter, the client complains of b. Attempt of the body to adjust or normal reflex
discomfort in the bladder and in the urinary mechanism
meatus. The initial action of the nurse would be: c. Fluid and electrolytes imbalance
* d. Irritation of the urinary bladder
1 point
30. When considering the safety needs of a client
a. Establish patency of the catheter with a urinary catheter, which of the following
b. Check the bladder if distended should the nurse observe?
c. Notify the physician *
d. Milk the catheter toward the collecting 1 point
receptacle
a. Irrigate the catheter daily
27. The client is ordered to have closed b. Keep the bag lower than the bed
intermittent catheter irrigation. The nurse c. Measure intake and output daily
performs the procedure in the following order: d. Keep a closed sterile drainage system

1. Aspirate sterile solution into the syringe SITUATION


ional Development is achieved through
participation in various trainings and continuing
2. Using aseptic technique, put sterile solution in
professional education. This is to keep nurses
the sterile graduated cup
abreast with the latest development in the
profession and staying globally competitive in
3. Clamp indwelling retention catheter skills development.

4. Withdraw syringe, leave solution for about 20 31. Nurse Palomeno is attending a
minutes cardiopulmonary resuscitation training to review
her previous CPR training as a requirement in the b. Make the professional globally competitive by
new hospital where she was recently employed. maintaining capability for delivering professional
This is an example of: services.
* c. Augment the nurse educational preparation for
1 point admission to the practice of his profession.
d. Make available latest trends in the profession
a. Continuing education brought about by scientific and technological
b. Advanced training advancement in the profession.
c. In- service training
d. Professional training SITUATION
A group of nursing students is conducting a
32. A nurse who recently passed the nurse research on the medical ward of a Laoag General
licensure exam who is seeking employment Hospital with the topic “The Effects of the Nursing
decided to attend training on IV therapy program Health Teachings in the Reduction of Anxiety of
offered by an accredited nursing organization. This Diabetic Patients Undergoing Limb Amputation.”
is an example of:
* 36. The group adviser will most likely suggest a
1 point study to check the procedures of the research. This
is:
a. Advanced training *
b. Professional training 1 point
c. Continuing education
d. In- service education a. Proposal
b. Test-retest
33. After a year and a half of working in a hospital, c. Pretest
Nurse Palomeno decides to pursue a master’s d. Pilot study
degree in nursing. Graduate education in nursing
prepares nurse Palomeno for the following except: 37. The independent variable of the study will be
* *
1 point 1 point
a. Take advance training as a clinical specialist a. Nursing health teachings
b. Assume managerial positions in nursing service b. Insulin injection
c. Carry out research to advance nursing theory c. Reduction of anxiety
d. Take lead roles in nursing educational settings d. Diabetic patients
34. Nurse Prince believes that health is a 38. Which of the following is the variable that can
fundamental right of every individual. He believes be measured?
in the worth and dignity of each human being and *
recognizes the primary responsibility to preserve 1 point
health at all costs. These statements are part of the:
* a. Reduction of anxiety
1 point b. Diabetic Patient
c. Nursing helath teachings
a. Philippine Nursing Act of 2002 d. Limb Amputation
b. Code of Good Governance
c. Standard of Nursing Practice 39. What research design is likely to be utilized?
d. Code of Ethics for Registered Nurses *
1 point
35. The objectives of continuing professional
education programs in nursing are the following a. Descriptive
except: b. Quasi-experimental
* c. Exploratory
1 point d. Experimental

a. Protect and promote the general welfare of the 40. Which of the following is an example of
public by attaining the lowest standards and convenience sampling?
quality in the practice of profession. *
1 point A. Ask for a report from the staff nurses regarding
the new protocol and its implementation
a. Subjects are purposively picked by the B. Call for a meeting with the staff to discuss
researchers problems encountered
b. Selection of every 10th person on a patient list C. Interview each staff separately to effectively
c. Simple selection identify areas for improvement in the protocol
d. Availability of subjects D. Search for more evidence regarding the new
SITUATION protocol and present in a meeting
Florence is a Charge Nurse of the Charity Pediatric 45. Which of the following statements should be
Ward. She is handling 45 patients for the morning avoided by Flo in giving feedback to the staff after
shift, with 3 RNs, 2 Nursing Attendants, and 1 Ward a performance appraisal?
Clerk. *
41. The staff are able to wish their off schedule with 1 point
Florence, thereby their motivation to please her all A. “Gather all the staff in the meeting room so I can
the time. What is her primary source of power over announce the results of the evaluation.”
the others in the unit? B. “Let us talk about how you can improve your
* techniques in administering medications.”
1 point C. “Go over this list of strengths and weaknesses we
a. Coercive power have identified during your evaluation.”
b. Informational power D. “If your weakness is in the computation of fluid
c. Reward power infusions, I will be assigning you that task for one
d. Expert power month so you can practice.”
Rationale: Never give feedback to them as a group.
42. Annually, Florence assists the unit’s Head Nurse
during budgeting. This activity is under what stage SITUATION
of management? In community Health Nursing the realities of
* “cause- effect” relationship almost always exist and
1 point the CH Nurse is in a very strategic position to
decipher these occurrences. The following
A. Planning questions apply.
B. Organizing
c. Staffing 46. In a political rally, spaghetti and friend chicken
d. Controlling were distributed among the 1,500 people who
attended. Around 2-3 hours after lunch, people
43. In the ward, the staff work as a team. Florence started experiencing gastrointestinal symptoms
is responsible for carrying out the orders of the such as stomach pains, nausea and vomiting,
doctors, while each of the nurses are assigned to diarrhea, fever and dehydration. Health authorities
feed and bathe the patients, administer due confirmed that the outbreak of illness was mainly
medications and therapeutics, and monitor vital due to staphylococcal enterotoxins. This condition
signs and fluid infusions. What nursing care is an illustration of a statistical association which is:
delivery system is utilized in the unit? *
* 1 point
1 point
a. Direct causal
A. Modular Nursing b. Non- causal
B. Team Nursing c. Indirect causal
c. Primary nursing d. Multifactorial
d. Functional nursing
47. The above illness outbreak describes which
44. A new protocol for administering tube feedings type of epidemic?
among neonates is introduced in the unit. The staff *
are having problems in the implementation. What 1 point
is Flo’s first action?
* a. Cyclic variation
1 point b. Point source
c. Secular variation 1 point
d. Propagated epidemic
a. pay particular attention to the economics status
48. This type of epidemic is characterized by the b. observe the family and the environment
following EXCEPT: c. put emphasis on the children’s education
* d. observe the family’s spiritual practices
1 point
53. In your community home visits, you have met
a. Epidemic terminates when supply of susceptible four clients. Who among them will you visit first?
persons is exhausted. *
b. Simultaneous exposure of the susceptible 1 point
persons
c. Cause originated from single event a. Family planning defaulters
d. Low number of susceptible personal, high b. A premature newborn
number of immune persons c. G4P3 3 days post partum
d. G3P2 36 weeks AOG
49. Here, the nurse would want to determine the
magnitude of the problem therefore she will 54. During a home visit, socialization is important
compute for: in order to:
* *
1 point 1 point

a. Prevalence rate a. Do procedures


b. Incidence rate b. Establish rapport and put the client at ease
c. Disease rate c. identify the needs of the client
d. Attack rate d. evaluate the visit

50. In cases like epidemics it is necessary for the 55. To give clear and correct information to the
nurse to be able to call to action and mobilize the client, the nurse should consider the following
community towards: EXCEPT:
* *
1 point 1 point

a. Community response a. Give several instructions at one time


b. Individual response b. Listen to what the patient is saying
c. Leaders response c. do not use medical terms
d. Public officials response d. repeat important information

SITUATION SITUATION
One of your responsibilities as a community health The Good Shepherd Parish Church in Las Piñas City
nurse is to conduct home visits to evaluate the initiated a parish led 5 year community health and
health condition of families and communities. development project. Community diagnosis is
needed to enable its parish public health nurse
51. Prior to the visit you have checked on your prepare the necessary community health nursing
clients’ records in the health center to have a care plans and programs which would be
background of their case and to formulate responsive to the needs of the parishioners and
objectives of the visit. This phase is: various communities. To realize all these
* collaborative work is necessary.
1 point
56. As the parish community health nurse working
a. closing with volunteer RNs and affiliate you will assess
b. planning community and environment characteristics.
c. professional Which of the following pertains to environmental
d. socializing characteristics?
52. In every home visit, you are not only concerned
1. language, religion and political orientation
with the client but you should also:
*
2. occupation, unemployment status and poverty 60. Evaluating the nursing care given, which of the
level following vital statistics in the communities served
indicates the BEST health status?
3. air, water and noise pollution *
1 point
4. vegetation and sanitation a. 0 infant mortality rate for the year 2006
* b. 0 crude death rate for the year 2007
1 point c. 50% Swaroop’s index for the year 2006
a. 1 and 2 d. 0 crude birth rate for the year 2007
b. 1 and 4 SITUATION
c. 3 and 4 People empowerment is an important purpose
d. 2 and 3 why Community Participatory Active Research
57. A step in the development of the community (COPAR) was created, it encourages the
health plan that answers the questions, what health community to generate community participation in
problems, health threats and health deficits exist development activities.
and how the community copes with these health 61. In COPAR, people of the community are being
problems is referred to as: prepared as managers of development programs in
* the future. All of the following but one are
1 point considered as principles of COPAR
a. Assessment *
b. Plan implementation 1 point
c. Plan formulation a. People, especially the most oppressed, exploited
d. Evaluation and deprived sectors are open to change, have the
58. Collaboration is strengthened by the parish capacity to change and are able to bring about
health team with the city health department and change
the various health centers of the city stool b. COPAR should lead to a self-reliant community
examination days were conducted where majority and society.
of the children were found to have ascaris. Which c. Community resources are identified and
among the following community nursing diagnosis mobilized for the poor, the powerless and the
will guide the nurse in setting strategies to address oppressed
the problems? d. COPAR should be based on the interests of the
* poorest sectors of the society
1 point 62. COPAR is people-based it is focused towards the
a. parasitism as a health threat powerless and the oppressed. Which
b. parasitism foreseeable crisis developmental approach is related to
c. malnutrition as a health deficit participatory?
d. parasitism as a health deficits *
1 point
59. The following nursing interventions address
the problem on parasitism, EXCEPT: a. Abandoning the traditional methods of doing
* things and must adopt the technology of industrial
1 point countries
b. Introduction of whatever resources are lacking
a. teach proper disposal of stools and stress that it in the community adopting technological
shouldn’t be used as fertilizer development
b. encourage handwashing before and after eating c. Immediate or spontaneous response to
c. treat patients with broad spectrum anthelmintic ameliorate the manifestation of poverty, especially
d. stress that vegetables should be thoroughly on the personal level
washed especially if eaten raw d. The process of empowering the poor so that they
can pursue a more just and humane society
63. The following statements do not relate to iii. Vitamin C
community development, but one?
* iv. Iodine
1 point
a. In participatory approach, then nurse must v. Protein
devotedly adhere to what people want *
b. In a peasant community where people are 1 point
fighting for land ownership, the nurse must not a. i, ii, iv
participate as this is not a health concern b. i, ii, iii, iv
c. Nurses must not join protests action as nurses c. ii, iii, v
should always be neutral at all times d. ii, iii, iv
d. If the people are not attending to the services
offered by the health staff, the team must reassess 67. Micronutrient supplementation of Vitamin A
the needs of the people aids infants and children in their growth and
development. How much Vitamin A should be given
64. Carrying out the planned activities involving to a 7-month-old infant?
maximum community participation is referred to *
as: 1 point
*
1 point a. 200,000 IU for one dose only
b. 200, 000 IU for two doses, with an interval of two
a. Community Organization days
b. Community assembly c. 100,000 IU for two doses, 1 month apart
c. Mobilization d. 100, 000 IU for one dose only
d. Integration
68. When should a pregnant mother start taking
65. Community organizing ends when the Vitamin A supplements?
community is already self-reliant. This signals that *
the community organizers are now ready to pull 1 point
out of the community because
* a. As early as 2 months
1 point b. During the 4th month of pregnancy
c. Anytime during the last trimester
a. It indicates that community organization is d. At the first onset of uterine contractions
finished
b. It will prevent dependency of the community 69. In providing teachings to the livelihood
c. Organizers can expand to other poor residents and mothers of a barangay in Cavite, you
communities explain to them that the SangkapPinoy Seal means
d. It can evaluate the outcomes of the programs the food:
*
SITUATION 1 point
Being the staunch leader in health in the country,
the Department of Health developed numerous a. Is free from contamination from E. coli bacteria
health programs over the years. These programs b. Underwent fortification with Vitamin A, iron, or
address the leading health problems or conditions both
of different populations. c. Can complete the daily nutritional requirements
of an infant
66. Various nutrition programs are implemented d. Has been inspected by the FDA and considered
for a reduction in the common nutritional safe to eat
deficiencies present in the country. Most programs
are structured to address the deficiencies in these SITUATION
nutrients, namely: Kylie Gardo Verzosa, public health nurse of
Barangay Niyugyog. You have just finished
i. Vitamin A implementing the programs for the identified
community health problems last month. You are
ii. Iron now preparing to check if the target goals and
objectives were met. The following questions Immunization for different diseases are important
apply. for health care workers who are frequently
exposed to different patients and environments.
70. Evaluation involves which of the following
processes? 75. Pre-exposure vaccination for rabies consists of
* how many doses?
1 point *
1 point
a. Exploration
b. Observation a. Three
c. A and C b. Four
d. Measurement c. Five
d. Six
71. Which of the following phases of the PRECEDE-
PROCEED Model correspond to the evaluation if 76. Student Nurse Nicole was doing her ocular
the objectives and sub-objectives have been met? survey in Barangay Loma when a dog suddenly
* attacked her and bit her. Mara’s school records
1 point show she had complete prophylaxis for rabies. How
many doses of rabies vaccine does she expect to be
a. Process Evaluation given to her afer the bite?
b. Impact Evaluation *
c. Outcome Evaluation 1 point
d. Effect Evaluation
a. Three doses – two doses right away, and the
72. Which of the following is evaluated by the other dose on the 3rd day
outcome evaluation? b. Two doses – one dose right away, and the second
* on the third day
1 point c. None, since the pre-exposure vaccines are
a. Objectives already enough.
b. Strategy Objectives d. Four doses – one dose right away, and the other
c. Sub-objectives doses on the 3rd, 7th, and 14th days
d. Goal 77. Which of these vaccines is not given orally?
73. Which of the following evaluation indicators is *
described by the statement, “what proportion of 1 point
those who need something are actually receiving a. MMR
it”? b. Polio vaccine
* c. Rotavirus vaccine
1 point d. Cholera vaccine
a. Accessibility 78. All of the following vaccines except one are
b. Efficiency examples of live attenuated vaccines. Choose the
c. Coverage exception.
d. Effort *
74. What is the ultimate goal of community health 1 point
nursing? a. Varicella vaccine
* b. Measles vaccine
1 point c. MMR
a. Community competence d. Tetanus
b. Community self-actualization 79. In administering BCG to a neonate, how much
c. Community organization should Nurse Nicole withdraw from the vial?
d. Community integration *
SITUATION 1 point
a. 0.1mL
b. 0.5mL c. Children-beneficiaries aged 3-18 must enroll in
c. 0.05mL school, and maintain an attendance of at least 85%
d. 1.0mL of class days every month
d. Children aged 0-5 must receive regular
SITUATION preventive health check-ups and vaccine
The Aquino administration initiated Pantawid
Pamilyang Pilipino Program or commonly called 84. Rona, G1P1, is on her 2nd postpartum day. She
4Ps as its flagship poverty alleviation program. asks Nurse Maja about the definition of exclusive
breastfeeding. Nurse Maja responds based on his
80. Which of these government agencies is/are knowledge that exclusive breastfeeding means:
partners in enrolling beneficiaries from different *
state universities and colleges? 1 point
*
1 point a. Giving the baby breast milk and drops or syrups
consisting of vitamins, mineral supplements, or
a. Philippine Association of State Universities and medicines only.
Colleges b. Giving the baby breast milk only. Drops or syrups
b. Department of Labor and Employment consisting of vitamins, mineral supplements, or
c. Commission on Higher Education medicines should not yet be given until the 6th
d. All of the above month of life
81. Which of these programs is not included in the c. Giving the baby breast milk and water only.
4Ps? d. Giving the baby breast milk and solid food only.
* SITUATION
1 point Nurse Jabee Dayandante is caring for the Aguilar
a. Enrollment of children in daycare, elementary, Family and is currently doing his first level
and secondary schools assessment of the family’s nursing problems.
b. Deworming of schoolchildren aged 6 to 14 years 85. Mrs. Aguilar was looking at the chance of having
old a baby for the incoming year. Nurse Jabee knows
c. Health check-ups for pregnant women and that in the event that Mrs. Aguilar becomes
children aged 0 to 5 pregnant, the family will have what type of family
d. None of the above health problem?
82. How often are deworming pills given to *
children aged 6-14? 1 point
* a. Health Threat
1 point b. Health Deficit
a. Once a year c. Wellness state
b. Twice a month d. Stress point
c. Twice a year 86. Mrs. Aguilar gave birth to a baby girl named
d. Once a month Hetty. It was the second child of the family and all
83. To be able to receive the subsidies in 4Ps, which of the couple’s attention was focused on their new
among these is not part of the conditions that offspring. Twirly, who’s the elder sister of Hetty, is
should be met by the household-beneficiaries? becoming jealous of her. This was noted by Mrs.
* Aguilar and she asked the advised of the nurse. As
1 point a nurse, what would be the most appropriate
nursing intervention for this scenario?
a. Parents or guardians must attend the family *
development sessions, which include topics on 1 point
responsible parenting, health, and nutrition
b. Pregnant women must avail pre- and post-natal a. Tell Mrs. Aguilar that it’s normal phenomena.
care, and be attended during childbirth by a faith b. Create a plan that would address the jealousy of
healer. Twirly together with Mr. and Mrs. Aguilar.
c. Provide Mrs. Aguilar with a pamphlet regarding
child rearing practices.
d. Advise Mrs. Aguilar to reprimand Twirly for her 91. The said novel coronavirus is believed to
bad attitude. originate from what animal?
*
87. Mr. Aguilar who is a 15 packs/year smoker, 1 point
approached Nurse Jabee and told him that he was
diagnosed by the Pulmonologist with COPD. a. Donkeys
Knowing the pathophysiology of COPD, Nurse b. Camels
Jabee knows that the disease is chronic in nature. c. Bats
Given the situation, what type of family health Option 4
problem is the family experiencing?
* 92. Which of the following clients bear the most
1 point risk of acquiring a severe disease if infected with
MERS-CoV?
a. Health Deficit *
b. Health Threat 1 point
c. Stress point
d. Wellness state a. An 18-year-old female client who has a history of
rheumatic heart disease.
88. Mrs. Aguilar found out that Hetty was not able b. A 26-year-old male who works as a call-center
to get her last dose of immunization. This problem agent.
of Mrs. Aguilar can be noted as what type of of c. A 32-year-old female client who is on her 5th
family problem? month of pregnancy.
* d. A 46-year-old male client who has diabetes.
1 point
93. Typical findings in patients infected with MERS-
a. Health Threat CoV include which of the following:
b. Health Deficit *
c. Stress point 1 point
d. Wellness state
a. cough
89. The following are ways is the best way to b. hypothermia
measure the skill of Mrs. Aguilar in caring for her c. bradypnea
new infant, Hetty? d. vomiting
*
1 point 94. Your relative who is currently working as an
OFW in the United Arab Emirates called you and
a. Demonstrating how to hold her new infant. asked how he can prevent acquiring the infection.
b. Asking her to identify the steps in swaddling her You are correct by instructing him to do the
infant. following, except:
c. Asking her to verbalize the different ways of *
measuring the normal growth and development of 1 point
Hetty.
d. Demonstrating how to handle her stress. a. “Practice general hygiene measures before and
after touching animals like camels.”
SITUATION b. “Avoid close contact with potential sick
Middle East Respiratory Syndrome is a viral individuals.”
respiratory disease caused by a novel coronavirus c. “Have yourself vaccinated.”
that was first identified in Saudi Arabia in 2012. d. “Ensure that animal products be cooked
thoroughly.”
90. The clinical spectrum of MERS-CoV can include
all of the following, except: SITUATION
* Documentation and management of the patient’s
1 point records are two important tasks of the nurse. The
following questions apply.
a. Asymptomatic
b. Severe acute respiratory disease 95. During the interview of a patient, she starts to
c. Mild respiratory symptoms moan and curl up due to pain. History revealed that
d. None of the above the pain occurs about an hour after taking black
coffee without breakfast. It has been occurring for iv. Have a colleague listen on extension.
three weeks now. This is recorded as:
* v. Write down only the medication order
1 point and the date and time that the order was given.
*
a. Claims to have abdominal pains after intake of
1 point
coffee unrelieved by analgesics
b. After drinking coffee, the client experienced a. ii, iii, iv, and v
severe abdominal pain b. i, iii, and v
c. Client complained of intermittent abdominal c. ii and iv
pain an hour after drinking coffee d. i, ii, iii, iv, v
d. Reported abdominal pain usually an hour after
drinking black coffee. Pain has been felt for three 99. The nurse decided to call the attending
weeks now. physician of one of his patients because latter
suddenly collapsed. Which of the following should
96. Which of the following entries in the Nurses avoid when giving a telephone report?
Notes is most appropriate? *
* 1 point
1 point
a. Include the client’s name and medical diagnosis.
a. “Large hematoma noted in the right arm.” b. Begin the report with the main reason for the
b. “The client appears to be happy.” telephone call.
c. “Provided oral care.” c. Specify that the nurse would like the attending
d. “Clear breath sounds. Suctioning performed, as physician to come and assess the client.
ordered and as needed.” d. Inform the physician of the pertinent changes in
the patient’s baseline data.
97. The nurse is writing down his progress notes.
Should a recording mistake occur, which of the 100. A physician orders 2000 ml of D5NSS to infuse
following should he do? over 12 hours. The drop factor is 15 drops per
* ml. Nurse Lykee sets the flow rate at how many
1 point drops per minute?
*
a. Erase the erroneous entry using correction fluid
1 point
and write down the new entry above the erased
entry. a. 42 drops per minute
b. Draw a single line through it and write his initials b. 35 drops per minute
or name above or near the line. c. 21 drops per minute
c. Use multiple lines to strike through the d. 50 drops per minute
erroneous entry then write his initials or name
above or near the lines.
d. Draw a single line through the erroneous entry
and write error above the said entry.
98. The attending physician of one of the patients
called to give a new order. The nurse should do
which of the following when receiving telephone
orders?

i. Write the order down on the patient’s


Kardex.

ii. Ask the prescriber to speak slowly and


clearly.

iii. Read the order back to the prescriber


using the abbreviations which he used.
PRE-INTENSIVE EXAM NP3 1 point
Upon discharge, the patient with Chronic a. Risk for infection
Obstructive Pulmonary Disease (COPD) requires b. Anxiety level
considerable patient and family teaching; c. The oxygen requirements
d. Fluid intake
1. A nurse instructs a client diagnosed with COPD
to use purse-lip breathing. The client inquires the 5. COPD patients may be taught the following
nurse about the advantage of this kind of breathing. pulmonary hygiene measures to improve clearance
The nurse answers that the main purpose of purse- of airway secretion, except:
lip is to: *
* 1 point
1 point
a. Postural drainage
a. Prevent bronchial collapse b. Complete bed rest
b. Strengthen the intercostals muscle c. Effective coughing
c. Achieve maximum inhalation d. Measure fluid intake
d. Allows air trapping
SITUATION
2. Nurse Albert teaches a patient about the use of Sandy a flower shop owner admitted in medical
respiratory inhaler. Arrange the steps in using an ward has been resuscitated and was transferred to
inhaler chronologically. the ICU. The Physician inserted a CVP line. She was
diagnosed of having Congestive Heart Failure
1. Press the canister down with your fingers as (CHF).
you breathe in
6. CVP monitoring could provide the following
information, but one:
2. Wait one minute between puffs if more than *
one puff is prescribed 1 point

3. Inhale the mist, hold your breath at least 5 to a. Vascular tone


10 seconds before exhaling b. Blood volume
c. Ability of the heart to receive and pump blood
4. Remove the cap and shake the inhaler d. Glomerular filtration rate
* 7. he normal CVP reading is:
1 point *
a. 4, 1, 2, 3 1 point
b. 4, 1, 3, 2 a. 8 – 12 cm of H2O
c. 3, 4, 2, 1 b. 13 – 17 cm of H2O
d. 1, 2, 3, 4 c. 18 – 25 cm of H2O
3. The physician prescribed monitoring closely of d. 4 – 10 cm of H2O
clients oxygen saturation of the blood. Which of the 8. In taking the CVP reading, the nurse knows that
following will you prepare? the stopcock shall be manipulated in a manner
* that:
1 point *
a. Electrocardiogram machine 1 point
b. Spirometer a. There is a communication between the client and
c. Pulse oximeter the manometer
d. Blood Pressure apparatus b. There is a communication between the
4. Patients suffering from COPD are taught to avoid manometer and the client and closed to the IV
shifts to temperature and humidity. It should be c. There is a communication between the IV and the
emphasized that heat increases body temperature client and closed to the manometer
and thereby raising the d. There is a communication between the IV and
* manometer and closed to the client
9. The patient is having volume ventilator. Which of 1 point
the following does not indicate that the client is
adequately ventilated? a. Serving in other countries and learning new and
* modern ways of doing nursing and sharing these
1 point back in the Philippines.
b. Progressing as nurse-generalist in a multitude of
a. Absence of hyper – and hypoventilation choice-practice settings to that of expert nurse-
b. Skin is normal in color practitioner also in choice-practice-settings
c. Blood pressure is normal c. Avoiding personal and professional stagnation by
d. Presence of neurologic signs updating and upgrading one’s self
d. Constantly upgrading one’s self through
10. Possible means of verifying proper placement advanced technological means and strategies
of Lanie’s catheter (CVP line)?
* 13. It is important to remember that while RNs
1 point value “job tenure” because the years in service spell
variety of experiences in nursing practice, it is far
a. Palpation more valuable to consider that tenure-years are
b. Auscultation nothing if these are not parallel with one’s
c. Chest X-ray personal-professional growth and maturity. This
d. Cystoscopy implies:
SITUATION *
Enrolling as nursing students taught you what the 1 point
nursing profession has in store for you and to a. Simply earning years of job-related service until
recognize that each one came from different we retire from service.
environs, different influences, different past and b. Extending assistance to our less-fortunate fellow
present. As you journey through nursing, you saw nurses.
yourselves transform “from the person you were” c. Progressive upgrading of competencies in terms
to the “aspiring nurse” you have become. Now that of knowledge, skills, attitudes, and values as
you have graduated and now taking your Nurse professional nurse.
Licensure Examination (NLE) there is only the d. Volunteering our services wherever needed.
“YOU, who is the nurse.”
14. We often give our best in caring but despite all
11. As an aspirant, a beginning nurse practitioner efforts, the reality of facing death is inevitable. Our
after your basic nursing education, the “YOU, who brand and core values of nursing will always
is a professional nurse” means: extend beyond the ordinary levels of promotive,
* preventive, curative, and rehabilitative care. This
1 point culturally-bound, Filipino values of nursing
A. I have simply fine tuned myself, my needs, my likewise needs to be nurtured:
wants, my idiosyncrasies, to fit in the profession of *
nursing. 1 point
B. The I in me and the nurse in me are two distinct A. Psychological care
identities that even my patients have to learn to B. Emotional care
respect. C. Spiritual care
C. I have simply retained my former self but D. Relational care
acquired the knowledge, skills, attitudes, and
values expected of a nurse. 15. It is important to not only enrich one’s mind
D. The person I am and the professional nurse I with progressive technical upgrades but equip
aspire to be have now developed into one Filipino one’s self with holistic personal and professional
Nurse. We are one and the same identity. development believing that:
*
12. As you progress in developing your nursing 1 point
competencies, you have to thread a career-path
according to the culture and design of Philippine A. we are also God’s angels of mercy on earth
Nursing. This means: B. we may also find real holism in the service we
* render
C. we and the beneficiaries of our care are made up b. pulse rate
of body, soul, and spirit and each component do c. blood pressure
have health needs intertwined d. respiratory rate
D. should we encounter terminal patients, we may
understand how to support them to their dying 20. Nurses must be aware that pain in MI may occur
stage without cause primarily during what time of the
day?
SITUATION *
Aneleise, 55, was admitted to the medical ward for 1 point
acute myocardial infarction (AMI) management.
a. anytime of the day
16. The patient called for the nurse and complains b. usually after a day’s work
of unrelieved chest pain. She verbalized, “I followed c. early at night before retiring
your instructions carefully. I already took 3 NTG SL d. early in the morning
tablets at 5-minute intervals from my pillbox. But
the pain is still there.” The nurse’s best action SITUATION
would be: A nurse admitted a female, 19-year old college
* student. Her chief complaints are fatigue,
1 point weakness, and sometimes dizziness. The patient is
plae. The admitting diagnosis is iron deficiency
A. Oxygenate the patient immediately anemia.
B. c. Get NGT SL tab from the E-cart and give to
patient 21. The nurse prepared the client for complete
C. Call the doctor and report infarction blood count (CBC) testing. The complete blood
D. Administer Morphine stat as ordered count is normal if the result is:

17. Aneleise’s admission assessment was done by 1. Red blood cells – 3.6 to 5.0 million/mm3
Nurse Faye. Which of the following descriptions
would the nurse consider as a classical pain of 2. Reticulocyte – 1.0% to 1.5% of total RBC
acute myocardial infarction?
*
1 point 3. Hemoglobin – 14 to 16.5% g/dL

a. pain radiates to the jaw, back, and left arm 4. Hematocrit – 37 to 47%
b. crushing mediastinal pain
c. sudden chest pain associated with activity 5. Hemoglobin – 6 to 9 g/dL
d. gnawing pain unrelieved by rest
18. Oxygen at 2-4 L/min via nasal cannula was 6. Hematocrit – 40 to 50 %
prescribed. Nurse Faye understands that the *
primary purpose of this order is to: 1 point
*
a. 1,2,3,6
1 point
b. 1,3,5,6
a. increase myocardial oxygen supply c. 1,2,3,4
b. decrease cardiac workload d. 1,2,4,5
c. reduce pain due to ischemia
22. After a thorough assessment and based on the
d. relieve difficulty of breathing
laboratory findings, the diagnosis of iron deficiency
19. Morphine sulfate was administered in anemia is confirmed. The client asks the nurse what
intravenous bolus to reduce pain and anxiety. is the role of iron in the body? The correct response
Which of the following vital signs should Nurse of the nurse is:
Faye monitor carefully to specifically *
determine cardiac responses? 1 point
*
a. iron prevents bleeding
1 point
b. iron gives the red color of our blood
a. temperature
c. the body cannot synthesize hemoglobin without 27. To confirm the diagnosis of cholecystitis, the
iron attending physician ordered the procedure that
can detect gallstones as small as 1 to 2 cm and
d. iron helps in the conduction of nutrients to the inflammation. Nurse Jade would prepare the client
body for which specific diagnostic procedure?
23. Which of the following food enhance absorption *
of iron? 1 point
* A. cholangiography
1 point B. gall bladder series
a. cereals C. oral cholecystogram
b. citrus fruits D. ultrasonography
c. dairy products 28. The diagnosis was confirmed as cholecystitis
d. green leafy vegetables with gallstones. The doctor prepared the client for
24. The client was prescribed Ferrous sulfate as the removal of his gallbladder. The client asks the
iron supplement. For better absorption, the nurse nurse: “How will this procedure affect my
would instruct the client to take this supplement: digestion?” The nurse’s most correct response
* would be:
1 point *
1 point
a. with meals.
b. 1 hour before meals. A. c. “Your body system will adjust in due time.”
c. after breakfast B. “The removal of the gallbladder usually
d. before going to bed interferes with digestion but can be remedied by
dietary modifications.”
25. Intramuscular supplementation of Iron causes C. “The removal of the gallbladder would
local pain and can cause stain in the skin. If you are significantly interfere only with the digestion of
the nurse, what the best technique of fatty food.”
administration will you use? D. “The removal of gallbladder does not usually
* interfere with digestion.”
1 point
29. While reviewing the laboratory findings of the
a. Z-track client, Nurse Jade found out that which findings are
b. IV bolus elevated?
c. vigorous rubbing of the injection site after
injection 1. white blood cell count
d. use the gluteus maximus muscle
SITUATION 2. total serum bilirubin
Nurse Jade is in charge of a client who was admitted
for management of acute episodes of cholecystitis. 3. alkaline phosphate

26. Nurse Jade did her admission assessment. She 4. red blood cell count
understands that the pain is characterized as:
*
5. cholesterol
1 point
A. Tenderness that is generalized in the upper 6. serum amylase
epigastric area *
B. Tenderness and rigidity at the left epigastric area 1 point
radiating to the back
C. Tenderness and rigidity of the upper right A. 3,5,6
abdomen radiating to the midsternal area B. 1,2,6
D. Pain of the left upper quadrant radiating to the C. 1,2,3
left shoulder D. 2,3,4
30. A T-tube was inserted and the doctor ordered: *
“Monitor the amount, color, consistency and odor 1 point
of drainage.” Which of the following procedures
can the nurse perform without the doctor’s order? a. “These supplements enhance the production of
* RBC.”
1 point b. “The vitamins and iron can suppress bone
marrow function.”
A. clamping c. “Actually, the patient does not need these
B. emptying supplements.”
C. aspirating d. “It is best that the client gets these supplements
D. irrigating from natural sources.”
SITUATION 35. The client complained of generalized pruritus.
A nursing student was assigned to take care of a The following are appropriate nursing
client who was diagnosed of polycythemia vera. interventions, except:
*
31. You planned the nursing care of the client 1 point
together with the nursing student. You asked the
nursing student to enumerate the clinical a. administer routine antihistamine round the clock
manifestations of a client with polycythemia vera. b. regulate room temperature to 25 degrees or
You expected the nursing student to enumerate the lower
following manifestations, except: c. bathe in tepid or cool water followed by coca-
* based lotion application
1 point d. wearing light material, loose-fitting camisa
a. splenomegaly SITUATION
b. ruddy complexion Nurse Carolina is assigned to care for clients in the
c. generalized pruritus medical ward for clients suffering from renal and
d. hepatomegaly urinary concerns.
32. The nursing student reviews laboratory 36. An adolescent is hospitalized with acute
findings and finds which blood results are glomerulonephritis (AGN). Which aspect of the
elevated? client’s history is most significant to her diagnosis?
* *
1 point 1 point
a. RBC, WBC, platelet count A. Sitting next to a student with mononucleosis
b. WBC, platelet and cholesterol B. Immunization with VZIG after exposure to
c. bilirubin, RBC and platelet varicella
d. BP, WBC, and hematocrit C. Experiencing a bout of cystitis within the past
year
33. Phlebotomy was ordered as part of the therapy. D. Having an untreated sore throat two weeks ago
You instructed the client and emphasized that the
procedure can be repeated. The client inquired, 37. A client with renal calculi has just returned
“What is the primary aim of the procedure?” Your from having extracorporeal shock wave lithotripsy
appropriate response is: (ESWL) to the right kidney. Following the
* procedure, the nurse should:
1 point *
1 point
a. “Remove the excess blood and donate to patients
of the same blood type.” A. limit the client’s fluid intake for the next eight
b. “Prevent headache and dizziness.” hours
c. “Keep the BP reading within normal range.” B. monitor the nephrostomy tube for kinking or
d. “Keep the hematocrit within normal range.” occlusion
C. strain the urine to detect any stone fragments
34. The companion asks why the client was advised D. apply anesthetic cream over the client’s right
to avoid iron supplements or vitamins. The correct flank
response of the nurse would be:
38. The nurse is providing dietary teaching for a *
client with a history of uric acid kidney stones. 1 point
Which food should be avoided by the client on a
purine-restricted diet? A. Rice
* B. Baked salmon
1 point C. Potatoes
D. Barbecued beef
A. liver
B. tapioca 43. The nurse notes that the latest potassium level
C. milk for a client in renal failure is 6.2 mEq. The first
D. peanut butter action by the nurse should be to:
*
39. The nurse is discharging a client with urethritis. 1 point
Which of the following would be most important
for the nruse to include in this client’s discharge A. Call the laboratory and repeat the test
instructions? B. Alert the cardiac arrest team
* C. Obtain an ECG strip and have lidocaine available
1 point D. Take the vital signs and notify the physician

A. warm sitz bath with bath salts 44. A client with acute renal failure moves into the
B. eliminate phosphorous-rich food diuretic phase after one week of therapy. During
C. daily application of a vaginal deodorant spray this phase the client must be assessed for sign of
D. avoid sexual intercourse until symptoms subside developing:
*
40. A nurse caring for a patient who ahs an ileal 1 point
conduit (ileal loop) following a cystectomy for
bladder cancer should teach the patient that the A. Renal failure
type of drainage expected from the stoma is: B. Hyperkalemia
* C. Hypovolemia
1 point D. Metabolic acidosis

A. urine and mucus shreds 45. A client with a transplanted kidney is taught the
B. clear urine signs of rejection. The nurse would know that the
C. urine streaked with blood teaching was effective when the client says that a
D. both urine and feces sign of rejection would be:
*
SITUATION 1 point
The nurse cares for patients with problems of the
urinary tract. A. An increased urinary output
B. A subnormal temperature
41. The client with chronic renal failure is on a C. An elevated blood pressure
restricted protein diet and is taught about high- D. Weight loss
biologic-value protein foods. An understanding of
the rationale for this diet is demonstrated when the SITUATION
client states that high-biologic-value protein foods Mrs. Jones, 45 years old, is a patient under your
are: care in the Pay Ward. She is a known hypertensive
* with poor compliance to her pharmacologic
1 point treatment. During your rounds, your VS of Mrs.
Jones are as follows: T 37.3C, HR 97bpm, RR 15
A. Necessary to prevent muscle wasting breaths/min, and BP 140/95. Your physical
B. Used to increase urea blood products assessment also showed that she has pitting edema
C. Needed to promote weight gain on her lower extremities.
D. Responsible for controlling hypertension
46. Because of the inability of the kidneys to filter
42. The home health care nurse visits a 40-year-old properly, accumulation of wastes occurs. Which of
housewife who has been placed on hemodialysis. these manifestations is not related to the
When receiving the diet with the client, the nurse electrolyte imbalances occurring in CKD?
encourages her to include: *
1 point 1 point
a. Trousseau’s sign A. Health educator
b. Calcification of major blood vessels B. Event coordinator
c. Tremors C. Researcher
d. U-wave D. Advocate for a healthy lifestyle
47. The patient may exhibit impaired secretion of 52. A female student asked you if she is overweight
aldosterone, which eventually may be manifested or not. What information would you solicit from
in: her for you to be able to answer her question?
* *
1 point 1 point
a. Anorexia A. her age
b. Jugular vein distention B. her height and age
c. Vomiting C. her height and weight
d. Sunken eyeballs D. her weight and age
48. Which of these drugs would cause an 53. She told you that she is 16 years old, weighs 176
interaction and serious effect with loop diuretics? lbs and she stands 5 ft. and 4 inches. Your answer
* to her would be:
1 point *
1 point
a. Atorvastatin
b. Digoxin A. "You are on the verge of being classified as
c. Clopidogrel obese"
d. Tramadol B. "You are overweight"
C. "You are underweight"
49. Lanthanum carbonate is given to the patient to D. "You are within the normal range"
address which electrolyte imbalance:
* 54. Another student asked you on the risk factor for
1 point Obesity. Using a psycho-biologic theory your
answer would be:
a. Hypocalcemia *
b. Hypernatremia 1 point
c. Hyperkalemia
d. Hyperphosphatemia A. Peer pressure and low self-confidence
B. Eating disorder as the results of living in a diet-
50. Low-dose dopamine was ordered for a patient conscious society
with Acute Kidney Injury. Which of these would C. Eating disorder as a generic trait
you monitor to measure its effectiveness? D. Eating disorder as a fixation at the oral stage
*
1 point 55. You were asked if Meridia, an appetite
suppressant, is safe to take. Your response was:
a. Heart rate *
b. Urine output 1 point
c. Capillary time refill
d. Respiratory rate A. "I will refer you to the dietician so she can
incorporate the drug in the diet plan for obese
SITUATION client"
You are a school nurse in the public school of B. "What information have you gathered regarding
Municipality of Cavite. As part of your Nutrition the drug?"
Program you will give a talk on "Nutritional C. "there are known adverse side effects of the drug,
problems affective Highschool Students." headache being the most common complaint"
51. As a school nurse, part of your responsibility is D. "I will refer you to the pharmacist so he can
as a/an: discuss further the action of the drug"
* SITUATION
Electrolyte and fluid imbalances normally occur in 60. Calcium gluconate is ordered for a patient
patients with renal and metabolic disorders. As a whose serum calcium falls under 8.6mg/dL. Which
nurse, it is your responsibility to be able to of the following actions would necessitate
differentiate normal from abnormal laboratory intervention?
values, findings, and patient responses. This way, *
you will be able to address efficiently and properly 1 point
the electrolyte imbalance(s) the patient is
presenting. a. Incorporating the calcium gluconate in sodium
bicarbonate
56. How does insulin work in treating b. Administering calcium gluconate via slow IV
hyperkalemia? bolus
* c. Monitoring the patient for postural hypotension
1 point d. Using D5W as a diluent for calcium gluconate
a. Insulin binds with potassium for excretion in the SITUATION
urine Alfonsus sought hospital confinement for pleuritic
b. Insulin causes the pituitary gland to produce pain, fever, and cough. The attending physician had
aldosterone to increase sodium and water a chest x-ray taken STAT. Result revealed presence
absorption and potassium excretion of lung infiltrates. The client was assigned to
c. Insulin causes potassium ions to be transported Kianne the staff nurse.
into the cell
d. Insulin increases the affinity of glucose to 61. When Kikay performed chest auscultation, she
potassium observed short discreet bubbling sounds over the
lower region of the right lung. Which of the
57. In caring for Lucy who has fluid volume deficit, following abnormal findings will Kikay consider?
which among these evaluations would indicate that *
the patient is recovering from the fluid imbalance? 1 point
*
1 point A. Friction rub
B. Murmur
a. Urine specific gravity 1.05 C. Wheezes
b. CVP 5 mmHg D. Crackles
c. Urine output 10mL/hr
d. Hematocrit 55% 62. Kikay put her priority nursing diagnosis as
“Ineffective airway clearance related to increased
58. What is the primary and initial manifestation of secretions and ineffective coughing.” Which
hypernatremia? nursing intervention would be considered to
* facilitate coughing with the LEAST discomfort?
1 point *
1 point
a. Increased DTRs
b. Delusions A. Splinting chest wall with pillow when coughing
c. Thirst B. Putting the client in semi-Fowler’s position all
d. Peripheral edema the time
C. Taking cough med q4 hours round the clock
59. A bipolar patient develops hyponatremia due to D. Utilizing the purse-lip technique of breathing
excessive vomiting. Which of these medications of
the patient could cause toxicity induced by 63. The physician prescribes oral penicillin 500 mg
hyponatremia? every six hours for seven days. On the fifth day,
* before Kikay administers the first dose for the day,
1 point she computed for the total amount in the
milligrams of the oral penicillin that has been
a. Valproate received by the client. Which of the following is the
b. Clozapine correct amount?
c. Lithum *
d. Risperidone 1 point
A. 2,500 mg D. Client focused, measurable and allowing the
B. 15,000 mg client to understand what needs to be done
C. 10,000 mg
D. 8,000 mg 68. The new nurse is asked to review the
medication list for a client who will undergo
64. Standard precaution dictates that the nurse pulmonary function test. The nurse manager asks:
observes which of the following when caring for a “Which of the following drug class should be
client with streptococcal pneumonia? withheld prior to the test”. The CORRECT answer of
* the new nurse is:
1 point *
1 point
A. Use of face mask
B. Use of sterile gloves A. Antiussives
C. Observe two-feet distance when giving care B. Bronchodilators
D. Use clean gloves C. Antiemetics
D. d. Antibiotics
65. Sputum cultures are to be obtained to establish
the client’s specific antibiotic treatment. Kikay 69. A physician orders a urine specimen for culture
would BEST collect the specimen: and sensitivity STAT for Pedro in room 102. The
* nurse manager delegated the function to the new
1 point nurse. Which of the following statements is the
MOST appropriate for the nurse manager to say to
A. Early in the morning initiate delegation of task?
B. Early morning after an antiseptic gargle *
C. After brushing the client’s teeth 1 point
D. Anytime of the day after a warm salt solution
gargle A. “We need urine for culture STAT for Pedro in
room 102. Tell me when you send it to the
SITUATION laboratory.”
A new licensed nurse is assigned to the medical B. “A STAT urine culture was ordered for client in
unit. The nurse manager assesses that nurses skill room 102. Would you get it please.”
is deficient. The following questions apply. C. “We need a STAT urine culture for Pedro in room
66. Which action is MOST appropriate for the nurse 102.”
manager to undertake/plan for this new nurse in D. “Please get the urine sample for culture and
her unit? sensitivity in room 102 client.”
* 70. Towards the end of the shift, the nurse manager
1 point noted increasing unrest among the staff due to the
A. Discuss with the new nurse her alternatives tardiness of the new nurse in finishing her charting.
B. Counsel the new nurse that if her performance Which action is BEST for the nurse manager to
doesn’t improve in 2 months, she will be replaced. take?
C. Refer the new nurse to the Human Resource *
department for re-evaluation 1 point
D. Plan with the new nurse to work on how to A. Dismiss the problem as nothing as it is not
improve her skills interfering with the ward activity
67. The nurse manager wants to assess B. Ignore the observation and proceeds to her office
improvement of the new nurse. She asks which of C. Discuss the observation with the staff
the following characteristics of client goal setting in D. Report to the nurse supervisor
the care plan is CORRECT. The goal should be: SITUATION
* In COPD, the airflow limitation is progressive and is
1 point associated with response of the lungs to noxious
A. Client focused, realistic and measurable particles or gases. Mr. De Leon has just been
B. Nurse focused, time-limited, realistic and admitted to the unit with a diagnosis of acute
measurable exacerbation of end-stage COPD.
C. Nurse focused flexible, measurable and realistic
71. Upon physical assessment, you noted the a. Complaints of chest tightness and difficulty of
following: RR 37breaths per minute, dyspneic, and breathing
dusky-colored skin. Which of the following should b. Crackles head upon auscultation of the lung
you perform initially? fields, and presence of red raised areas on the chest
* c. Symmetrical expansion of the thoracic cavity
1 point during inspiration
d. Temperature reading of 37.8C with feelings of
a. Monitor the client’s ECG tracing nausea
b. Attach Mr. De Leon to an oxygen source via nasal
cannula regulated at 10lpm SITUATION
c. Position the patient to High-Fowler’s Anne Grace. 45 years old post-menopausal,
d. Contact the doctor immediately consulted the physician in the OPD for abdominal
pain. She has been stressed, irritable lately. She is
72. Which assessment finding by the staff nurse in more relaxed when she smokes and drinks
a client with COPD would require the supervisor to alcoholic beverages. She was advised by the doctor
intervene? to be admitted for a suspected duodenal ulcer.
*
1 point 76. Upon admission, the nurse obtained further
history of the patient. Which of the following is a
a. Presence of barrel chest and dyspnea typical pain pattern of a patient with duodenal
b. Oxygen flow meter set at 2lpm while the client is ulcer?
ambulating *
c. Rust-colored specimen collected early in the 1 point
morning
d. Use of accessory muscles during inspiration A. Pain is felt after 1-2 hours after meals
B. Burning, cramping pressure like pain
73. Mr. Basilio, a 43-year-old client, is currently C. Pain is not relieved with antacids
prescribed with Propanolol for his hypertension. D. Burning in the high left epigastrium, back and
As you browsed through this patient’s chart, you upper abdomen
noted history of bronchial z take first before
administering Propanolol? 77. Ms. Anne was ordered to have blood typing. The
* nurse is aware that the higher incidence of
1 point duodenal ulcer belongs to what blood type?
*
a. Measure the apical pulse rate for a full minute 1 point
b. Instruct the patient to change position slowly
c. Contact the physician to verify order A. A
d. Give drug with meals B. 0
C. B
74. Upon receiving a client with status asthmaticus, D. AB
your priority intervention should include:
* 78. Gastrointestinal endoscopy was ordered in
1 point order to have better visualization of the gastric
mucosa. After the procedure, the patient developed
a. Start infusion of Theophylline IV high grade fever of 39.8 degrees Centigrade with
b. Monitoring of breath sounds every 5-1o minutes acute abdominal pain. What PRIORITY nursing
c. Give the client a short-acting beta adrenergic intervention should the nurse perform?
agonist *
d. Administer Solu-medrol, a glucocorticoid, IV 1 point
75. A nurse is completing the admission A. Assess the patient at once for possible
assessment of a patient with Reactive Airway perforation
Disease. Which manifestation would indicate to the B. Take vital signs every two hours
nurse that the patient has such condition? C. Give tepid sponge bath
* D. Give relief medication as ordered
1 point
79. Which of the following conditions is 83. You just finished your pre-operative teaching to
contraindicated for nasogastric tube insertion and Mr. Vergilio. Which of the following statements
need not be ordered by the physician? Patients indicates that further teaching is needed?
* *
1 point 1 point
A. With cranio-facial surgery A. "The pre-operative medications will allay my
B. All of the options fear of the operation"
C. With hypophysectomy B. " The breathing exercises will help expand my
D. Recovering from gastric surgery lungs after surgery
C. " I will try to ambulate after the anesthesia has
80. Which of the following is an EXPECTED worn off'
outcome in a patient with duodenal ulcer? The D. " I am glad I can rest and be relaxed of my office
patient will work"
*
1 point 84. Which of the following assessment would be
your indication that Mr. Vergilio has recovered
A. Join friends for social drinks every weekend from anesthesia. A sense of?
B. Engage self with smoking cessation program *
C. No longer join sports program 1 point
D. Use Tylenol for control of pain
A. Feeling hungry
SITUATION B. Feeling of warmth in the room
Mr. Vergilio, a lawyer has been under medical C. Pin pricks in the toes
treatment for multiple gastric ulcer for the last 5 D. Going to the toilet
years and has been advised surgery by his doctor.
85. After Bilroth II surgery, Mr. Vergilio reported
81. The patient has to undergo a Bilroth II surgical that he was having diarrhea after meals. What
operation. Which of the following is done in this should the nurse recommend to prevent this
procedure? condition:
* *
1 point 1 point
A. A partial removal of the distal 2/3 of the stomach A. Take antacids before mealtime
and anastomosis of gastric stump to the jejunum B. Avoid eating saturated fats
B. A partial removal of the distal 2/3 of the stomach C. Avoid taking fluids
and anastomosis of the stump to the duodenum D. Observe fasting overnight
C. Removal of the large portion of the stomach and
the pyloric sphincter SITUATION
D. Removal of the whole stomach Maxine, 35 years old, consulted the OPD because of
pain and abdominal distention. She also
82. Prior to surgery, you were asked by Mr. Vergilio complained of frequent urination in small amount.
why atropine sulfate is ordered, your BEST Doctor's impression is presence of kidney stone.
response is:
* 86. The client is asked to describe the pain she is
1 point experiencing. Which of the following should the
Nurse expect? 1. Intense deep ache in the costo-
A. "Atropine causes your laryngeal spasm and will vertebral region 2. Pain usually radiates anteriorly
inhibit relaxation of the muscle" and towards the bladder 3. Wavelike pain that
B. "Anesthetic agents can increase mucus secretion radiates down to the thigh and genitalia 4. Acute
and atropine will counteract this effect" excruciating pain, colicky in nature that radiates to
C. "Anesthetic agents can cause bradycardia and genitalia
atropine will counteract this effect" *
D. "Atropine relaxes the spinal cord and will relieve 1 point
the pain
A. 1 and 2
B. 1 only
C. 2 and 3 indigenous barangay health workers view their
D. 1,2,3 and4 work as grass root professionals, the team is using
what ethnographic perspective?
87. Which of the following questions should the *
Nurses ask to determine the PREDISPOSING factor 1 point
of stone formation?
* a. Etic perspective
1 point b. Emic perspective
c. Tacit perspective
A. How much water do you drink everyday? d. Exo perspective
B. Do you engage in outdoor sports?
C. What kind of medications do you take? 92. Who developed the interpretive
D. Do you have a family history of stone formation? phenomenology methodologic school of thought?
*
88. A 24-hour urine collection was ordered for 1 point
Maxine. This is a measurement of the following,
EXCEPT: a. Husserl
* b. Parse
1 point c. Heidegger
d. Glaser and Strauss
A. Calcium
B. Creatinine 93. Which of the following phenomenological
C. Magnesium analytic methods require a validation of results by
D. Sodium returning to the study participants?
*
89. Lithotripsy was ordered for Maxine, She asked 1 point
the Nurse "What is this procedure all about? The
nurse replied, "It: A. Van Kaam
* B. Colaizzi
1 point C. Giorgi
D. Strutfest
A. Will break up the stones in the calyces of the
kidney" 94. Coombs (2007) explored the phenomenon of
B. Is a procedure to extract renal stones" spousal caregiving from the perspective of the
C. Will create electrical discharge to break the spousal caregivers (the lived experience of spousal
stones in the kidney" caregivers). The research question of the study
D. Will visualize the stone and destroy it" was: “What is it like for older caregivers to care for
a spouse who has survived a stroke?” What type of
90. Maxine is preparing to go home. Which of the research was done by the author?
following is NOT an essential component of your *
health instruction? 1 point
*
1 point A. Phenomenology
B. Ethnographic study
A. Engage in outside activity so she can sweat a lot C. Grounded theory
B. Drink 8-10 glasses per day D. Feminist research
C. Restrict intake of protein to 60 g/day
D. Take non steroid drugs for pain 95. What is the correct order of preparing
qualitative data before analysis?
SITUATION
Conrad, a newly-graduated nurse, is currently
i. Transcribing Qualitative Data
working as a Research Assistant with his former
professor on a qualitative study regarding the
health education strategies used by barangay ii. Managing/ Organizing Qualitative Data
health workers.
iii. Developing a Category Scheme
91. A qualitative study is undertaken by the
research team. If the study focuses on how iv. Coding Qualitative Data
* C. Drainage on burnt areas
1 point D. Paralysis
A. III, I, IV, II 100. Overresuscitation is not uncommon during
B. I, III, IV, II the first days following burn treatment. Which of
C. III, I, II, IV these signs may indicate overresuscitation?
D. III, II, I, IV *
1 point
SITUATION
A 56-kg patient is rushed to the ER due to scald A. Shortness of breath
burns. It was noted that he had full-thickness burns B. Urine output of 1L/day
on his right arm, right thigh, half of the right leg; C. Temperature of 38.5C
and partial-thickness burns on his chest and D. Increased lethargy
abdomen in front. You are assigned to care for the
patient.
96. During the resuscitation phase, which of these
do you expect the nurse to do?
*
1 point
A. Administer insulin to treat hyperglycemia.
B. Start furosemide therapy to increase potassium
excretion.
C. Start clear-liquid diet to increase nutrient intake.
D. Starting the patient on an isotonic crystalloid-
infusion for resuscitation.
97. What should the nurse monitor first to
determine the effectiveness of the fluid challenge?
*
1 point
A. Skin turgor
B. Urine output
C. Heart rate
D. IV site
98. A nurse is about to administer an H2 blocker for
the patient. She knows that this medication
addresses which complication of burn injury?
*
1 point
A. Pruritus of the affected areas
B. Loss of gastric motility following burn injury
C. Severe pain caused by the exposed nerves on
burn areas
D. Gastric mucosal sloughing from loss of plasma
volume
99. Which of these manifestations is an early
indication of compartment syndrome?
*
1 point
A. Absent capillary refill
B. Pain with passive ROM
PRE-INTESIVE EXAM NP4 nurses do
*
Nurses working on the medical unit are 1 point
experiencing difficulties stabilizing the clients’
nasogastric tubes. In order to offer the clients the a. Review all the studies, summarize findings, and
best choice of care, the nurse must do data completely revise NGT stabilization policy
gathering. b. Revise practice-based guidelines and procedures
of clinical intervention
1. Before data gathering, which of the following c. Gather enough evidences to substantiate the
should be initiated by the nurse? findings of the nurses in the medical unit
* d. Conduct a similar research in the ICU
1 point
SITUATION
a. Develop the question or problem statement that The nurse’s accurate assessment is very crucial in
best defines the need preventing complication during the severe post
b. Begin with a concept and seek how to best burn period.
implement the concept into client care
c. Secure consent from the clients for their 6. Corazon sustained circumferential thermal
involvement burns of the left upper extremity and chest. You
d. Organize a team to conduct the study and obtain noted that pulse could not be appreciated in his
their comment injured extremity. Which of the following will you
do first?
2. The next step that the nurses should do is: *
* 1 point
1 point
A. elevate the injured extremity to increase blood
a. Analyze the extent of literature flow to the heart
b. Define the problem B. remove the dead tissues which impede
c. Secure evidence circulation
d. Formulate framework C. try to take the pulse in the uninjured extremity
3. Following the review and evaluation of the D. notify the physician immediately as this requires
data, the nurses should: emergency intervention
* 7. While loading containers with chemicals from a
1 point truck, Shoogar accidentally spilled the whole can of
a. Recommend measures to meet the need corrosive chemicals all over his body. Which of the
b. Extrapolate information following would you consider as the priority
c. Choose the most relevant information intervention in the emergency management of
d. Decide the level of evidence Shoogar.
*
4. With the information obtained to promote 1 point
evidence-based practice, the nurse should:
* A. wash the chemical off with cool water
1 point B. maintain a patent airway
C. assess for associated injuries
a. Apply findings to clients with NGT D. remove all clothing containing the chemicals
b. Conduct a pilot study to selected clients in
several units of the study 8. A 165-lbs trauma client was rushed to the
c. Make recommendation and apply into NGT emergency with full thickness burns of the whole
practices in the unity face, right and left arm, and at the anterior chest
d. Communicate findings to nurses in other units sparing the abdominal area. He also has superficial
partial thickness burn at the posterior trunk and at
5. Following the success in stabilizing NGT in their the half upper portion of the left leg. In caring for
unit, the nurses share the literature they found on the patient during the emergent phase of burn,
NGT stabilization with nurses in the ICU. In order using the Parkland’s formula, you know that during
to reflect currently published knowledge, which of the first 8 hours of burn; the amount of fluid that
the following appropriate actions should critical will be given is:
* 12. To confirm his impression of colorectal cancer,
1 point Hans will require which diagnostic study?
*
A. 5,400 mL 1 point
B. 10,600 mL
C. 9,450 mL A. carcinoembryonic antigen
D. 6,750 mL B. stool hematologic test
C. abdominal computed tomography (CT) test
9. Janine, who has partial thickness burns on the D. proctosigmoidoscopy
face, inquires about skin care after discharge.
Which of the following should you include in your 13. The following are risk factors for colorectal
discharge teaching? cancer, except:
* *
1 point 1 point
A. Avoid facial makeup for at least a year A. inflammatory bowels
B. Continue to eat high caloric food for the next B. high fat, high fiber diet
month C. smoking
C. Wear a pressure garment daily for one year D. genetic factors-familial adenomatous polyposis
D. Avoid sunlight for the next three months
14. Symptoms associated with cancer of the colon
10. You are caring for Leo who sustained severe include:
burn injury and he is in the emergent phase of burn *
injury. As his nurse, you gathered the following: 1 point
hemoglobin 13.5g/100mL., hematocrit 50%,
serum Na 130 mEq/L. How will you explain the A. constipation, ascites and mucus in the stool
laboratory results? B. blood in the stools, anemia and pencil-shaped
* stools
1 point C. diarrhea, heartburn and excruciating pain
D. anorexia, hematemesis, and increased persitalsis
A. They are slightly abnormal but will normalize
once IV fluids have been started 15. Several days prior to bowel surgery, Hans may
B. These are due to hemodilution from rapid IV be given sulfasuxidine and neomycin primarily to:
fluid replacement *
C. All the laboratory tests are within normal range 1 point
D. These are due to a loss of serum and interstitial A. reduce the bacterial content of the colon
fluid through the burn wounds B. promote rest of the bowel by minimizing
SITUATION persitalsis
Colorectal cancer can affect old and younger C. soften the stool by retaining water in the colon
people. Surgical procedures and other modes of D. empty the bowel of solid waste
treatment are done to ensure quality of life. You are SITUATION
assigned in the Cancer Institute to take care of Infection can cause debilitating consequences
patients with this type of cancer. when host resistance is compromised and
11. Hans, 55 years old who is suspected of virulence of microorganisms and environmental
colorectal cancer, is admitted to the clinic. After factors are favorable. Infection control is one
taking the history and vital signs, the physician important responsibility of the nurse to ensure the
does which test as a screening test for colorectal quality of care.
cancer? 16. Tonnie, who has been complaining of anorexia
* and is always feeling tired, develops jaundice. After
1 point a work-up, he is diagnosed of having Hepatitis A.
A. barium enema His wife asks you about gamma globulin for herself
B. carcinoembryonic antigen and her household help. Your most appropriate
C. annual digital rectal examination response would be:
D. proctosigmoidoscopy *
1 point
A. “Don’t worry, your husband’s type of hepatitis is As a member of the health and nursing team, you
no longer communicable.” have a crucial role to play on ensuring that all
B. “Gamma globulin provides passive immunity for members participate actively on the various tasks
Hepatitis B.” agreed upon.
C. “You should contact your physician immediately
about getting gamma globulin.” 21. While eating his meal, Matt accidentally
D. “A vaccine has been developed for this type of dislodges his IV line and bleeds. Blood oozes on the
hepatitis.” surface of the over bed table. It is most appropriate
that you instruct the housekeeper to clean the table
17. Greg develops a nosocomial respiratory tract with:
infection. He asks you what that means. Your *
response would be: 1 point
*
1 point A. acetone
B. alcohol
A. “You acquired infection after you have been C. ammonia
admitted to the hospital.” D. bleach
B. “This is a highly contagious infection which
requires complete isolation.” 22. You are a member of the infection control team
C. “You should contact your physician immediately of the hospital. Based on a feedback during a
about getting gamma globulin.” meeting of the committee there is an increased
D. “this type of infection usually resolves on its incidence of pseudomonas infection in the burn
own.” unit (3 out of 10 patients had positive blood and
wound culture). What is your priority activity?
18. As a nurse, you know that one of the *
complications that you have to watch out for when 1 point
caring for Fritz who is receiving total parenteral
nutrition is: A. Establish policies of surveillance and monitoring
* B. Assign point persons who can implement
1 point policies
C. Do data gathering about the possible sources of
A. stomatitis infection (observation, chart review, interview)
B. hepatitis D. Meet with the nursing group working in the burn
C. infection unit and discuss problem with them
D. dysrhythmia
23. Part of your responsibility as a member of the
19. A solution used to treat pseudomonas wound diabetes core group is to get referrals from the
infection is: various wards regarding diabetic patients needing
* diabetes education. Prior to discharge today, 4
1 point patients are referred to you. How would you start
prioritizing your activities?
A. Dakin’s solution *
B. half strength hydrogen peroxide 1 point
C. acetic acid
D. betadine A. Determine their learning needs then prioritize
B. Bring your diabetes teaching kit and start your
20. Which of the following is most reliable in session taking into consideration their distance
diagnosing wound infection? from your office
* C. Contact the nurse-in-charge and find out from
1 point her the reason for the referral
A. purulent drainage from a wound D. Involve the whole family in the teaching class
B. culture and sensitivity 24. You have been designated as a member of the
C. WBC count of 20,000/µL task force to plan activities for the Cancer
D. gram staining Consciousness week. Your committee has 4 months
SITUATION to plan and implement the plan. You are assigned
to contact the various cancer support groups in 28. Which of the following can be used on the
your hospital. What will be your priority activity? irradiated skin during a course of radiation
* therapy?
1 point *
1 point
A. clarify objectives of the activity with the task
force before contacting the support groups A. adhesive tape
B. find out if there is budget for this activity B. mineral oil
C. determine the VIPs and celebrities who will be C. talcum powder
invited D. Zinc oxide treatment
D. Find out how many support groups there are in
the hospital and get the contact numbers of their 29. Earliest sign of skin reaction to radiation
president therapy is:
*
25. You are invited to participate in the medical 1 point
mission activity of your alumni association. In the
planning stage, everybody is expected to identify A. desquamation
what they can do during the medical mission and B. erythema
what resources are needed. You thought it is also C. atrophy
your chance to share what you can do to others. D. pigmentation
What will be your most important role where you 30. What is the purpose of wearing a film badge
can demonstrate the impact of nursing in health? while caring for the patient who is radioactive?
* *
1 point 1 point
A. conduct health education on healthy lifestyle A. identify the nurse who is assigned to care for
B. take the initial history and document findings such patient
C. be a triage nurse B. prevent radiation-induced sterility
D. act as a coordinator C. protect the nurse from radiation effects
SITUATION D. measure the amount of exposure to radiation
As a nurse in the Oncology Unit, you have to be SITUATION
prepared to provide safe, efficient and effective In a disaster, there must be a chain of command in
care to your patients. place that defines the roles of each member of the
26. Which one of the following nursing response team. Within the health care group, there
interventions would be most helpful in preparing are pre-assigned roles based on education,
the patient for radiation therapy? experience, and training on disaster.
* 31. As a nurse, to which of the following groups are
1 point you best prepared to join?
A. offer tranquilizers and antiemetics *
B. instruct the patient of the possibility of radiation 1 point
burn A. transport group
C. emphasis on the therapeutic value of the B. treatment group
treatment C. triage group
D. map out the precise course of treatment D. morgue department
27. What side effects are most apt to occur to 32. There are important principles that should
patient during radiation therapy to the pelvis? guide the triage team in disaster management that
* you have to know if you were to volunteer as part
1 point of the triage team. The following principles should
A. urinary retention be observed in disaster triage, except:
B. abdominal, vaginal, or perineal discharge *
C. paresthesia of the lower extremities 1 point
D. nausea, vomiting, and diarrhea
A. any disaster plan should have resources B. “Improvement of the processes in a proactive,
available to triage at each facility and at the disaster preventive mode is paramount.”
site if possible C. “A chart audits to find common errors in practice
B. do the greatest good for the greatest number of and outcomes associated with goals.”
casualties D. “A flow chart to organize daily tasks is critical to
C. make the most efficient use of available the initial stages.”
resources
D. training on disaster is not important to the 37. The nurse manager identifies that time spent by
response in the event of a real disaster because staff charting is excessive, requiring overtime for
each disaster is unique in itself completion. The nurse manager states that “staff
will form a task force to investigate and develop
33. Which of the following categories of conditions potential solutions to the problem, and report on
should be considered first priority in a disaster? this at the next staff meeting.” The nurse manager’s
* leadership style is best described as:
1 point *
1 point
A. intracranial pressure and mental status
B. lower gastrointestinal problems A. participative
C. respiratory infection B. laissez-faire
D. trauma C. autocratic
D. group
34. A guideline that is utilized in determining
priorities is to assess the status of the following, 38. The nurse manager informs the nursing staff at
except: morning report that the clinical nurse specialist
* will be conducting a research study on staff
1 point attitudes toward client care. All staff are invited to
participate in the study if they wish. This affirms
A. perfusion the ethical principle of:
B. locomotion *
C. respiration 1 point
D. mentation
A. participativeness
35. Induction of vomiting is indicated for the B. autonomy
accidental poisoning patient who has ingested: C. anonymity
* D. justice
1 point
39. The hospital has sounded the call for a disaster
A. aspirin drill on the evening shift. Which of these clients
B. rust remover would the nurse put first on the list to be
C. gasoline discharged in order to make a room available for
D. toilet bowl cleaner new admission?
SITUATION *
Quality management is essential in nursing and has 1 point
been a critical part of nursing administration. As A. A middle-aged client with a history of being
nurses, it is important to know by heart the ventilator dependent for over 7 years and admitted
concepts related to such with bacterial pneumonia five days ago
36. Which statement made by a nurse about the B. An elderly client with a history of hypertension,
goal of total quality management or continuous hypercholesterolemia and lupus, and was admitted
quality improvement in a health care setting is with Steven-Johnson Syndrome that morning
correct? C. A young adult with diabetes mellitus type 2 for
* over 10 years and admitted with antibiotic-induced
1 point diarrhea 24 hours ago
D. An adolescent with a positive HIV test and
A. “It is to observe reactive service and product admitted for acute cellulites of the lower leg 48
problem solving.” hours ago
40. A triage nurse has these 4 clients who arrived 44. Barbara, the old woman was now immobilized
in the emergency department within 15 minutes. and brought to the emergency room. The x-ray
Which client should the triage nurse send back to shows a fractured femur and pelvis. The ER nurse
be seen first? would carefully monitor Barbara for which of the
* following sign and symptoms?
1 point *
1 point
A. A teenager who got a singed beard while
camping A. Tachycardia and hypotension
B. A middle-aged client with intermittent pain B. Fever and bradycardia
behind right scapula C. Bradycardia and hypertension
C. A 2 month old infant with a history of rolling off D. Fever and hypertension
the bed and has bulging fontanels with crying
D. An elderly client with complaints of frequent 45. Which of the following serious complications
liquid brown-colored stools can occur with long bone fractures?
*
SITUATION 1 point
Manolo, a registered nurse, witnessed an old
woman hit by a motorcycle while crossing a train A. Bone emboli
railway. The old woman fell at the railway. Monolo B. Fat emboli
rushed at the scene C. Platelet emboli
D. Serous emboli
41.As a registered nurse, Manolo knew that the first
thing that he will do at the scene is: SITUATION
* Postoperative infection complication is still a
1 point concern in surgical client’s care. Hospital staff
needs to review practices to adhere to the
A. Stay with the person and encourage her to standards of care to improve quality and safe care
remain still and immobilize the leg while waiting delivery.
for the ambulance
B. Leave the person for a few moments to call for 46. Nurse Alona is setting up for an emergency
help Cesarean Section. The linen packs were damp
C. Reduce the fracture manually although these were just taken from the sterilizer.
D. Move the person to a safer place The nurses’ appropriate action is:
*
42. Manolo suspects a hip fracture when he noticed 1 point
that the old woman’s leg is:
* A. Bring the linen packs back to the central supply
1 point section for quality control
B. open the linen pack and allow to dry
A. Lengthened, abducted and internally rotated C. change the damp linen pack
B. Shortened, abducted and externally rotated D. do not use the damp linen
C. Shortened, adducted and internally rotated
D. Shortened, adducted and externally rotated 47. The clinical instructor assigned a nursing
student to assist in the operation. When the nursing
43. The old woman complains of pain. Agua noticed student entered the OR suite, her curly long hair
that the knee is reddened, warm to touch and was not completely covered by the head cap. What
swollen. Manolo interprets that this signs and would the circulating nurse do?
symptoms are likely related to: *
* 1 point
1 point
A. assist the nursing student to tuck-in all her hair
A. Infection inside the head cap
B. Inflammation B. welcome the nursing student to the OR
C. Thrombophlebitis C. request the clinical instructor to tell the nursing
D. Degenerative disease student to use the head cap properly
D. do not allow the nursing student to scrub in
48. After the surgeon finished doing the surgical B. gastrojejunostomy
hand scrub, she came into the OR suite swinging C. enterostomy
her hands casually. The scrub nurse would do D. esophagojunostomy
which of the following appropriate action?
* 52. After admitting the client to the PACU, the first
1 point action of the nurse should be:
*
A. offer a sterile towel to dry her hands 1 point
B. serve the surgeon her sterile gown and gloves as
usual A. assess patency of the airway
C. tell the circulating nurse to pour alcohol 70% to B. assess the client’s pain
the surgeon’s hands C. monitor vital signs
D. remind the surgeon to scrub again D. check the rate of the IV infusion

49. When the intern-in-charge did the skin prep 53. As the nurse monitors the client, she notices a
and catheterized the client, the circulating nurse bright red spot on the dressings which measure 4
noticed when the intern withdrew the catheter cm in diameter. The nurse would initially do which
from the vagina. What is your appropriate and appropriate nursing intervention?
immediate action? *
* 1 point
1 point A. notify the client’s surgeon of a potential
A. stop the intern and do the catheterization hemorrhage
yourself B. assess for presence of drainage
B. offer to change the catheter C. change the top dressing
C. alcoholize the tip of the catheter before D. continue to monitor the vital signs
reinserting the catheter 54. In assisting the client to do deep breathing,
D. hold the hand of the intern to stop him from coughing and turning to the sides on the first
reinserting the catheter postoperative day, which nursing action would be
50. After the last stitch, the surgeon is ready to most helpful for the client?
apply dressing to the incision wound. Which of the *
following does the nurse expect the surgeon to do? 1 point
* A. restate the importance of respiratory exercises
1 point B. give the client reassurance that he can cough,
A. remove his gloves and apply the dressings breathe deeply, and turn to sides safely
B. apply the dressings and tape and then remove C. administer the prescribed analgesics round the
his gloves clock
C. put the dressings and remove his gloves to apply D. apply abdominal splint while coughing
the tape 55. The client complained of abdominal pain,
D. tape the dressings and remove his gloves nausea and vomiting, with abdominal distention.
SITUATION The nurse anticipates which of the following
A 55 year old male client, Herald, post Billroth II priority management after referring to the
was admitted to the post anesthesia care unit surgeon?
(PACU) from the OR. The client is still sedated but *
responsive to commands. He has nasogastric tube 1 point
(NGT) draining orange-yellow fluid to a drainage A. possible surgery
bottle. B. endoscopy
51. The nurse who admitted the patient recognizes C. rectal tube insertion
that Billroth II procedure means: D. gastric decompression
* SITUATION
1 point Nurse Mak admitted patientLouie for management
A. gastroduodenostomy of acromegaly.
56. Nurse Mak is aware that acromegaly is a 1 point
condition when growth hormone occurs in excess
in adulthood or after epiphyses of the long bones A. alteration in fat and carbohydrate metabolism
have fused. The following are the typical physical B. changes in blood sugar level
features of the disorder, except: C. changes in appearance
* D. alteration in their voice
1 point SITUATION
A. the soft tissues continue to grow A 35 year old female client, Claire is with Grave’s
B. hands and feet are enlarged disease was admitted for treatment. The physician
C. client grows taller prescribed Prophyl Thraci (prophylthiouracil) to
D. broad and bulbous nose treat the disorder.

57. The client was prescribed Ocreotide acetate 61. Before the nurse administers the medication,
(Sandostatin). Nurse Jayne would monitor for which of the following is most relevant for the
which of the following side effects? nurse to ask?
* *
1 point 1 point

A. dysuria A. “Do you experience gastric discomforts?”


B. hypotension B. “Do you prefer a liquid form of medication?”
C. abdominal pain C. “When was the last time you took alcohol?”
D. constipation D. “When was your last menstruation?”

58. For effective dosing, Ocreotide acetate must be 62. The client is prepared for surgery in about 10
administered by which appropriate route three days time. Lugol’s solution 4 gtts P.O. was
times weekly? prescribed for 10 days. The client asked the nurse
* for the purpose of the drug. Which response of the
1 point nurse is correct?
*
A. intravenously 1 point
B. subcutaneously
C. orally A. It eliminates the need to take hormone
D. intramuscularly replacements
B. It decreases the risk of bleeding
59. Priority discharge plans should include which C. It decreases the risk for thyroid crisis
of the following? Select all that apply. D. It stabilizes your immune system to withstand
surgery
1. fasting blood sugar
63. When the client returns to the unit after
surgery, which technique is most appropriate to
2. bone assessment monitor bleeding from the incision?
*
3. intake and output 1 point

4. urine output A. remove the dressings to directly inspect the


* incision wound
1 point B. weigh all gauze dressing before and after
changing wound dressings
A. 2 and 3 C. assess for dampness at the back of the client’s
B. all except 1 neck
C. 1,2 and 3 D. pass a flashlight
D. 1 and 2
64. Which of the following assessment findings
60. Acromegaly often develops insidiously that when observed in a post thyroidectomy client is
nurses should understand that the client with indicative of a thyroid crisis?
disorder would seek medical care because of: *
* 1 point
A. spasm in the hand 3. DOH Administrative Order and CHED
B. falling blood pressure Memorandum Order
C. regular and noisy respiration
D. high fever 4. House Bills/ Senate Bills
*
65. At the start of thyroid replacement post total
1 point
thyroidectomy, the nurse must monitor for side
effects. Which side effects would the nurse expect A. All are recognized sources
to assess? Select all that apply. B. 1 and 4
C. 2 and 3
1. hypertension D. 1 and 4
68. The following are true about the requirements
2. tremors
for applying to take the Licensure Examination in
the Philippines except:
3. hirsutism
1. The applicant must be a person of Good Moral
4. insomnia Character

5. tachycardia 2. The applicant must have gotten an average of


75% with no grade lower than 60% in any subject
6. hyperglycemia in the NLE
*
1 point 3. The applicant must have finished a BSN degree
A. 1,2,4 and 5 from an accredited school
B. 3,4,5 and 6
C. 1,2,3 and 5 4. The applicant must be a Filipino citizen
D. 1,3,4 and 5 *
1 point
SITUATION
It is often said that “ignorance of the law excuses no A. 1and 3
one.” In the practice of professional nursing the B. 1and 4
same applies.The following questions are related to C. 1, 3, and 4
ethico- legal dimension in the practice of nursing. D. only 2

66. For failure of the nurse to check the nasogastric 69. ASEAN countries now engage in Mutual
tube placement before administering feeding, the Recognition Arrangement (MRA) negotiations
patient aspirated and died. The nurse may incur: consistent with liberalization measures of the
* General Agreement on Trade in Services (GATS).
1 point The applicable mode for this kind of agreement is:
*
A. Administrative liability 1 point
B. Civil liability
C. All liability may be filed A. Via Reciprocity
D. Criminal liability B. Via Examination
C. Via Temporary/ Special permit
67. The following are sources of D. Via Movement of Natural Persons
laws/rules/policies that affect the nursing practice
in the Philippines: 70. Upon full implementation of Board of Nursing
Resolution No.22, series of 2009 (National Nursing
1. International Council of Nurses/ Philippine Career Progression Program), the following are
Nurses Association Advisories deemed legally practicing nursing except:

2. Board of Nursing Promulgations e.g. Resolutions 1. A “trained” nurse phlebotomist


and Memorandum
2. A duly certified hemodialysis nurse who primes 1 point
a hemodialysis machine before initiating
hemodialysis to a client a. Nothing by mouth for 10-12 hours
b. Blood extraction upon waking up
c. Instruct the client not to get out of bed until the
3. An ACLS- trained and Certified Nurse clinician
test is over
who intubates a patient under accepted ICCU
d. A good night sleep of 8-10 hours
protocol
74. Cytomel (Liothyronine) was prescribed to
4. A certified Critical Care Nurse Specialist who Lucille. Before administering the drug, Nurse Karen
extracts arterial blood every hour for ABG should not only verify the doctors order but also:
monitoring of patients *
* 1 point
1 point
a. Take the BP and PR
A. only 1 b. Take the temperature and respiratory rate
B. only 4 c. Take the weight of the client
C. only 3 d. Advise the client to stay in bed
D. only 2
75. Lucille asked the nurse which diet is most
SITUATION appropriate for her. The correct response of the
Nurse Katelyn admitted a 33 years old, married, for nurse would be:
thyroid work-up. *
1 point
71. Which admission assessment would make you
suspect that Lucille has hypothyroidism? Select all a. Low salt, low fat
that apply. b. Low calorie and high fiber
c. High calorie and high fiber
(1) Heat intolerance (4) Coarse and dry skin d. High protein but low residue
SITUATION
(2) Diarrhea (5) Somnolence Jojo and Jan Jan, a scrub nurse and circulating nurse
respectively, are preparing for hydrocelectomy,
(3) Bradycardia (6) Decreased appetite their last case for the day.
*
1 point 76. Jojo discovered a cut in her palm while she was
opening the sterile packs, Jojo should:
a. 2,3,5,6 *
b. All except 5 1 point
c. 3,4,5,6
d. 1,2,3,4 A. Scrub but put on double gloves
B. Ask to be relieved as a scrub nurse
72. Lucille was ordered to undergo radioactive C. Scrub if the cut is properly bandaged
iodine uptake the following day. The nurse D. Scrub after writing an incident report
understands that this test determines which of the
following? 77. Before any member of the surgical team
* proceed to do the surgical hand scrub, he/she
1 point should have complete operating room (OR) attire.
Identify all the attire appropriate for this case:
A. Absorption of the iodine isotope
B. Ingestion of the iodine isotope (1) Head Cap Goggles (3) Sterile gown
C. Stimulation of the iodine isotope
D. Reaction of the iodine isotope
(2) Face mask (4) Gloves
73. If the doctor will prescribe Basal Metabolic *
Rate (BMR) to Lucille, which of the following is NOT 1 point
indicated?
A. 2,3,4
*
B. 1,2,3
C. 1,2 ii. Sexual contact with males whose partners
D. 3,4,5 have had cervical cancer
78. A sterile set up has been prepared. The OR was
iii. Underweight
notified of a delay in transporting the client from
the ward to the OR. Which of the following
guidelines should the circulating nurse follows? iv. Folic acid deficiency
*
1 point v. Exposure to HPV6

A. Keep door of the operating room closed all the vi. Never had any sexual contact
time to maintain the “sterile set up” *
B. “Sterile set up” should be replaced after an hour 1 point
C. Cover appropriately “sterile set up”
D. Prepare another “sterile set up” a. i, iii iv, v
b. ii, iv
79. Identify which appropriate gloving technique c. ii, iii, iv, v, vi
will the assistant surgeon use when he performs d. i, iii, iv, vi
the skin preparation?
* 82. Cervical cancers metastasize if not detected
1 point and treated. What is commonly the first metastatic
location of cervical neoplasms?
A. Gloving self-closed technique *
B. Scrub nurse serves the gloves 1 point
C. Any gloving technique is accepted
D. Gloving self-open technique a. Inguinal lymph node
b. Cervical node
80. The intern 2nd assistant surgeon c. Pelvic lymph nodes
contaminated his gown while the surgery is d. Axillary lymph nodes
ongoing. He is expected to change his gown and
gloves. Which of the following is the CORRECT 83. A surgery is suggested to treat the patient’s
technique to be followed? cervical cancer. However, she exclaims that she still
* plans to bear a child. Which of the following
1 point procedures may be done to the patient granting her
wish?
A. The intern removes his gown and gloves then *
puts on another sterile gown and gloves. 1 point
B. The circulating nurse unties the gown. The intern
removes his gown then removes the gloves and a. Radical hysterectomy
puts another sterile gown and gloves. b. Radical vaginal hysterectomy
C. The intern removes his gloves, then his gown c. Radical trachelectomy
does a 3 minutes hand scrub and don another d. Simple bilateral salphingoectomy
sterile gown and gloves
D. The intern unties his gown removes his gown 84. Norma underwent hysterectomy and was
and put on another gown and gloves. transferred to your ward 3 hours post-surgery.
Which of these assessments would warrant you to
SITUATION notify the doctor?
Poon Day, who has an increased risk for cervical *
cancer asks you different points about the 1 point
condition.
a. Pain on the incision site
81. With your knowledge of cervical cancer, which b. Chest pain
of these would you include in your teaching on its c. Vaginal bleeding
risk factors? Select all that apply. d. Post-operative edema

i. Monogamous 85. Early ambulation for a patient who underwent


surgery for cervical cancer is encouraged to
prevent:
* c. “My whole body is itching terribly.”
1 point d. “Anything that touches my skin causes extreme
pain.”
a. Severe hemorrhage
b. Contractures 90. Which of these interventions would you not
c. Deep vein thrombosis include in caring for the patient?
d. Bladder retention *
1 point
SITUATION
A patient who has Stevens - Johnson syndrome is a. Shift to nasogastric gavage from IV infusions in
newly-admitted to the medical ward. You are fluid administration as soon as possible.
assigned to take the history and plan for the care of b. Prescribed topical antibiotic may be applied in
the patient. conjunction with hydrotherapy in a tub.
c. Ask the patient to limit blinking and eye
86. While you take your history, which of these movement to prevent aggravating the
signs and symptoms would be part of the complication.
prodromal stage of the syndrome? d. Administer analgesics before the painful
* procedures or treatments are done.
1 point
SITUATION
a. Dysuria You are a nurse in Indonesia who has been
b. Sore throat assigned as part of the triage team in the recent
c. Corneal lesion earthquake in the capital city. You assess and
d. Vaginal stenosis prioritize patients according to their status and
87. Which of these statements by the patient needs. Different emergency nursing concepts apply
would give you an information of the cause of the during this process.
syndrome? 91. Which of these should be done during your
* secondary survey of the patients?
1 point *
a. “I am not sure if I am able to drink at least 8 1 point
glasses of water because of my busy schedule.” a. Cervical spine stabilization
b. “I am fond of eating grilled barbecue and isaw b. Establishing a patent airway.
after work. I buy and eat them almost everyday.” c. Splinting of suspected fractures
c. “I take penicillin whenever I feel like I will have d. Immediate closed reductions on pulseless
cough or colds.” extremity
d. “Paracetamol is my go-to medication, especially
when I started feeling weak.” 92. In a reversed advanced type of triage, whom
would you give prioritization to?
88. In reviewing the workup of the patient, which *
of these laboratory findings do you expect her to 1 point
manifest?
* a. A gasping patient, with estimated blood loss of
1 point 900mL
b. An elderly client with a fractured femur, mobile,
a. Elevated arterial blood pH can respond to voice command
b. Elevated C-reactive protein c. A head trauma patient, unconscious, HR 40bpm,
c. Decreased partial thromboplastin time RR 10 cycles/min, BP 145/70
d. Decreased platelet count d. A pulseless infant, with cold and clammy skin
89. Which of these complaints by the patients 93. Critical incident stress management is what
indicate a progression to a complication of SJD? type of prevention in disaster management?
* *
1 point 1 point
a. “I have a difficulty swallowing liquids.” a. Primordial
b. “My eyes feel like they are drying.” b. Secondary
c. Primary d. Adjust the head of the bed to a flat position or as
d. Tertiary low as the client can tolerate
94. Following the external triage system, which 97. Using an overhead trapeze for repositioning the
among these patients would you highly prioritize? client can be accomplished by instructing the client
* to grasp the:
1 point *
1 point
a. An unresponsive patient with capillary refill time
of more than 2 seconds, and an RR of 10 a. Overhead trapeze with one hand and push with
breaths/min the heels upward
b. A pregnant client, with rib fractures, responsive b. Overhead trapeze with both hands and lift and
to voice commands, HR 120bpm, RR 20cycles/min pull during the move
c. A patient with sustained head injuries, gasping c. Head of the bed with one hand and maneuvering
for air, with bluish appearance for an upward movement
d. A patient with severe blood loss, PR of 75 bpm, d. Head of the bed with one arm and the overhead
RR of 10 cycles/min trapeze with the other arm then lift and pull
upward
95. Which of these principles is not included in
performing START triage in emergencies and 98. A client on bed rest is rolled to a lateral position
disasters? by the nurse. The nurse is negotiating the move
* correctly when he:
1 point *
1 point
a. Never go against the flow and go back to a
previous casualty a. Positions himself at the mid part of the bed and
b. Tag under the black category those persons who places both hands at the back of the client and roll
are gasping for air client onto side
c. Provide treatment for all patients requiring - Pull/roll the client toward you to the lateral
intensive care position
d. Begin where you stand. b. Places one hand on the client’s far hip and the
other on the client’s far shoulder, rock backward
SITUATION and roll onto side of the body facing him
Bed rest is a therapeutic intervention that achieves c. Assume a broad stance with the foot nearest the
beneficial effect. However prolonged bed rest can bed placing his arms under the client’s thighs and
be counterproductive to the clients’ recovery. The shoulder and roll client onto side
inactivity imposed by bed rest may cause structural d. Supports the back and buttocks of the client and
changes in joints and shorten muscles. Moving, shifts his own weight from the forward to the
turning, and positioning of clients are essential backward foot and roll the client onto side
aspect of nursing care.
99. A client with injured left leg is sitting on the
96. Nurse Monilia is giving the 8:00am medication bed preparing to transfer to a wheelchair. The
to a client who happens to have slid down the bed nurse is assisting the client and positions the wheel
from the fowler’s position. Which of the following chair on the:
interventions is most effective when the nurse *
repositions the client? 1 point
*
1 point a. Foot part of the bed
b. Client’s right side
a. Raise the head of the bed to the height of the c. Head part of the bed
center of gravity – Raise the BED to the height of the d. Client’s left side
center of gravity
b. Remove all pillows then place against the head of 100. A client has difficulty walking and needs a
the bed wheelchair to facilitate performance of daily
c. Ask the client to flex the hips and knees and activities. Anticipating the needs of the client, the
position the feet for effective pushing up nurse should have the wheelchair ready by placing
it at;
*
1 point
a. 80- degree angle
b. 45- degree angle Rationale: Lessen the amount of
body rotation required
c. 90- degree angle
d. 30- degree angle
PRE-INTENSIVE EXAM NP5 A. Emotional state
B. Sensorium or orientation
Facundo Mercado, a 35 years old male client C. Characteristics of talk
diagnosed with Delusional Schizophrenia is on D. Content of thought
pharmacotherapy as part of his medical
management 5. For proper documentation and accountability of
all entries to the client’s chart, it is important for
1. An order of Chlorpromazine (Thorazine) 300mg nurse Gigi to inspect that:
QID was given to the client. As a responsible nurse, *
Louisiana is reviewing the nurse’s notes. Which 1 point
assessment would alert her as most serious side
effect of this drug? A. Staff must not abbreviate SOAP
* B. Client’s problem in the medical record must bear
1 point date of entry and numbers of client’s problems
C. All notes must have signatures and title of the
A. Postural hypotension person making the entry
B. Photosensitivity and skin rashes D. Nurses implement the use of problem oriented
C. Inhibition of ejaculation and decreased libido progress notes
D. Agranulocytosis and jaundice
SITUATION
2. Which of these written notes by the nurses on the Ricardo Ricarding, 35 years old has been admitted
kardex would need further discussion on nursing to the psychiatric unit because he has been
actions related to antipsychotic medications negligent of his personal hygiene, has withdrawn
administration? himself from relating with others and seemed to
* have a world of his own as he was observed taking
1 point to himself.
A. Give parenteral meds only when the patient is 6. During the nurses rounds, nurse Annie observed
properly restrained that Manny lying on his bed , in a fetal position,
B. Teach the patient about the expected covered face with foul smelling body odor. Nurse
extrapyramidal side effects Annie noted this behavior as a state of:
C. Monitor the patient’s blood pressure *
D. Check to make sure that the client does not hide 1 point
medications
A. Disturbed affect
3. With the administration of Chlorpromazine, the B. Apathy and indifference
client complains of dry mouth and constipation. C. Autism
Which appropriate intervention should nurse D. Severe regression
Louisiana write in the nursing care plan?
* 7. In order to establish trust with Ricardo
1 point Ricarding, which of the following approaches is
most important to consider?
A. Question the client about the amount and type of *
his daily exercise 1 point
B. Consult with the client’s physician about
changing the antipsychotic medications A. Assign to him the friendliest nurse in every
C. Advice the client to chew sugarless gum and interaction
eliminate gas forming food B. Assign to him a male nurse in every interaction
D. Encourage the client to rinse mouth with water C. Have a nurse who portrays as a parent figure
and drink 6-8 glasses of fluid each day relate with him
D. Have the same nurse interact with him in every
4. During Nurse- Patient interaction, the client interaction
states that ants are crawling all over his body, you
would document this in which part of mental 8. Ricardo Ricardingremained aloof for several
status? days and the nurse have difficulty relating to him
* because he was often agitated, incoherent and
1 point
irrelevant in his thoughts. Medication was started, *
which medication will the psychiatrist prescribed? 1 point
*
1 point A. Impaired social interaction
B. Ineffective individual coping
A. Lithium anti manic C. Impaired Adjustment
B. Chlorpromazine antipsychotic D. Anxiety Moderate
C. Lorazepam anxiolytics
D. Sertraline SSRI 12. Obsessive compulsive disorder is BEST
described by:
9. Nurse Narcissa is conducting a health teaching to *
Ricardo Ricarding about the newly prescribed SSRI 1 point
antidepressant for the treatment of depression.
Which of the following points should Nurse A. Uncontrollable impulse to perform an act or
Narcissa include in her teaching plan? ritual repeatedly:
B. Recurring unwanted and disturbing thoughts
alternating with a behaviour.
1. These drug acts to increase the levels of mood
C. Pathological persistence of unwilled thought,
elevating chemical in the brain called serotonin
feeling or impulse
D. Persistent thoughts
2. These drug acts quickly, so feelings of depression
will decrease in few days 13. The defense mechanism used by persons with
obsessive compulsive disorder is undoing and it is
3. Manny should avoid alcohol or taking best described in one of the following statements:
antihistamine medications with this drug *
1 point
4. If Manny finds it difficult to sleep at night during A. Transfer of emotions associated with a particular
intake of the medication, he can try taking it in the person, object or situation to another less
morning threatening person, object or situation.
B. Unacceptable feeling or behaviour are kept out
5. Manny should have a special diet of awareness by developing the opposite behaviour
* or emotion.
1 point C. Consciously unacceptable instinctual drives are
diverted into personally and socially acceptable
A. 1,2,3 channels
B. 1,2,4,5 D. Something unacceptable already done is
C. 1,3,4 symbolically acted in reverse.
D. 1,4,5
14. To be more effective, the nurse who cares for
10. Given a client like Ricardo Ricarding, which of persons with obsessive compulsive disorder must
the following is a common nurses behavior during possess one of the following qualities:
the initial phase of Nurse- Patient Relationship *
which nurse Narcissa should be aware of: 1 point
*
1 point A. Consistency
B. Patience
A. Ambivalence C. Friendliness
B. Apathy D. Compassion
C. Withdrawal
D. Mistrust 15. Person with OCD usually manifest:
*
SITUATION 1 point
Hugeen seeks psychiatric counseling for his
ritualistic behavior of counting his money as many A. Fear
as 10 times before leaving home. B. Apathy
C. Suspiciousness
11. An initial appropriate nursing diagnosis is: D. Anxiety
SITUATION D. tell them that you are not upset or angry because
Last November 8, 2013, Supertyphoon Haiyan they did not talk
(Yolanda) left behind a path of destruction on one-
third of the Philippines, claiming many lives and 20. Rehabilitation of children after a disaster may
causing unimaginable damages never been seen include all except:
before. *
1 point
16. The following are the characteristic
manifestation of post traumatic stress disorder A. letting the child to be close to adults who are
except: familiar to them
* B. organize story telling session , singing songs and
1 point games
C. avoid touching, hugging and reassuring them
A. anhedonia verbally
B. extreme attachment with other people D. involve them in activities like drawing and
C. unresponsiveness to surroundings painting where they can express their emotions
D. flashbacks
SITUATION
17. All but one of the following is not a diagnostic Meniere’s disease is more common in adults in
criteria for the diagnosis of PTSD except their 40s, with symptoms usually beginning
* between the ages of 20s and 60 years. It appears
1 point equally common in both gender and most clients
have a familial history of meniere’s disease.
A. repetitive , intrusive recollection of reenactment
of the event in memories 21. A 42-year-old woman, Milagros with a
B. numbing feeling diagnosis of Meniere’s disease is seen in the clinic.
C. day time imagery or dreams The nurse would expect the client to complain of:
D. onset after 6 months of a traumatic event *
1 point
18. When the nurse is dealing with clients with
PTSD, which of the following approaches is A. Discharge from the ear, pain, and conductive
inappropriate? deafness.
* B. Vertigo, tinnitus, and neurosensory hearing loss.
1 point C. Fever, ear noises, and headache
D. Severe headache, enlarged lymph nodes, and
A. consistent empathic approach to help the clients fever
tolerate the emotional pain
B. simple reorienting , reassuring statements to 22. The nurse is assessing a patient diagnosed with
prevent suicidal ideation Meniere’s disease. Which of the following patient
C. trusting relationship to convey a sense of statements would require further teaching by the
respect, acceptance of their distress and belief in nurse?
clients’ reactions *
D. promote and maintenance dependence and the 1 point
clients’ highest level of functioning
A. “I smoke one pack of cigarette per day.”
19. For victims who refuse to talk whether angry, B. .“When I have vertigo, I keep my eyes open and
or remain mute and silent, the nurse should do the stare straight ahead.”
following except: C. “I have a continuous, low-pitched roar in my left
* ear.”
1 point D. d,“I continue to feel dizzy after the vertigo goes
away.”
A. maintain regular contact and greet them
B. acknowledge that you understand they are not to 23. The nurse finds a patient with Meniere’s
blame disease leaning over the sink in the room and
C. tell them you will not return to him so he should clutching it with both hands. After determining that
as well speak up the patient is having an acute attack, which of the
following actions should the nurse take FIRST?
* 1 point
1 point
A. Bloody drainage
A. Help the patient back to bed and place a pillow B. Yellowish drainage
on either side of the patient’s head. C. Greenish drainage
B. Have the patient lie down where he is and check D. Foul – smelling drainage
the patient’s vital signs and pupil’s response to
light. 28. Neurologic assessment should be performed
C. Give the patient an emesis basin and massage the every hour after the procedure to monitor an
neck over the area of the carotid arteries. increased intracranial pressure. Using the Glasgow
D. Notify the physician and prepare to administer coma scale, which of the following score indicates
atropine sulfate subcutaneously. that the client has the best neurological function?
*
24. During an acute attack of Meniere's disease, the 1 point
nurse can most likely anticipate administering
which of the following drugs? A. Eye opening-3, motor response-8, verbal
* response-6
1 point B. Eye opening-5, motor response-4, verbal
response-8
A. Corticosteroids C. Eye opening-4, motor response-6, verbal
B. Nonsteroidal anti-inflammatory drugs response-5
C. Antihistamines D. Eye opening-6, motor response-5, verbal
D. Diuretics response-4
25. What important client teaching should the 29. The nurse must stay alert for signs and
nurse provide regarding the client’s diet? symptoms of increased intracranial pressure (ICP).
* Which cardiovascular findings are late indicators of
1 point increased ICP?
*
A. Instruct to avoid foods rich in protein 1 point
B. Teach the client to read food labels
C. Instruct the client to limit salt intake at 2 grams A. Rising blood pressure and bradycardia
per day B. Hypotension and bradycardia
D. Encourage to have a high fiber diet C. Hypotension and tachycardia
D. Hypertension and narrowing pulse pressure
SITUATION
A 50 years old male client arrived in the emergency 30. To evaluate the extent of the increasing
room with complaints of frequent headaches. Left intracranial pressure, the nurse assesses the client
side body weakness and difficulty in balancing. An for Doll’s eyes reflex. When the nurse turns the
MRI was conducted and result shows a brain client head to the left, the client eyes remain in the
tumor. right side. This may indicate:
*
26. The patient has undergone a supratentorial 1 point
craniotomy to obtain tissue sample for biopsy.
What immediate nursing action should be done A. Brainstem Compression
after the procedure? B. Subdural hematoma
* C. meningeal inflammation
1 point D. Normal
A. Lying flat in bed SITUATION
B. Neurologic checks and vital signs every 4 hours Freddie, 35 years old driver, experienced fatigue,
C. Limiting fluids to 1.5-2 L in 24 hours visual disturbances and paresthesia in his arms and
D. Allowing no pillows under the head legs. He later developed spastic paralysis of the
legs. His physician says he has Multiple Sclerosis.
27. Drainage on a craniotomy dressing must be
measured and marked. Which of the following 31. Upon assessment, the nurse should address
should be reported immediately to the doctor? which of the following behaviour related with the
* disease?
* Parkinson’s disease is a slowly progressing
1 point neurologic movement disorder that eventually
leads to disability. Although the cause of most cases
A. Client’s coping is unknown, research suggest several causative
B. Client’s lifestyle factors, including genetics, atherosclerosis,
C. Client’s plan for the future excessive accumulation of oxygen free radicals,
D. Client’s actual and potential needs- holistic viral infections, head trauma, chronic use of
approach (includes option A, B and C) antipsychotic medications and some
32. The client is concerned about his fluctuating environmental exposure.
physical condition and generalized weakness. 36. Parkinson’s disease, a progressive neurologic
Which of the following is the priority nursing disorder is characterized by:
intervention for Freddie? *
* 1 point
1 point
A. Bradykinesia
A. Teach measures for activity limitation B. Tremors
B. Space activities throughout the day C. Muscle rigidity
C. Have an immediate family member stay with him D. All of the above
D. Bedrest and restriction of activities
37. The client is on an anti-Parkinsonian
33. During exacerbation of multiple sclerosis, the medication which acts by releasing dopamine from
client will experience which of the following? the neuronal storage sites. This anti-viral agent
* used early in Parkinson’s treatment is:
1 point *
A. Mental retardation 1 point
B. Resting Tremors A. Artane
C. sudden burst of energy B. Benadryl
D. diplopia and nystagmus C. Elavil
34. Mr. Freddie experiences bladder incontinence. D. Symmetrel
Which of the following should the nurse do? 38. The nurse will administer benztropine 1mg p.o.
* daily. Which finding suggests its desired side
1 point effect?
A. Limit fluid intake to 1200ml per day- *
2000ml/24hrs 1 point
B. Insert an indwelling catheter A. decreased confusion
C. Establish a regular voiding schedule B. decreased muscle treamors
D. Administer prophylactic antibiotic as ordered C. decreased dizziness
35. The nurse is preparing Manny for discharge D. decreased muscle rigidity
from the hospital. Which of the following is an 39. The client finds the resting tremor he is
appropriate instruction? experiencing in his right hand very frustrating. The
* nurse will advise this client to;
1 point *
A. Keep active, less stressful activities and avoid 1 point
fatigue A. take a warm bath
B. Learn to use walking aids in anticipation of B. practice deep breathing
future disabilities C. c, hold an object
C. Observe Quiet, inactive lifestyle and regular D. take diazepam as needed
exercise
D. Maintain good health habits and regular exercise 40. A patient with Parkinson’s disease has a
nursing diagnosis of Impaired Physical Mobility
SITUATION related to neuromuscular impairment. You observe
a nursing assistant performing all of these actions. a. Allow the client to have visitors and telephone
For which action must you intervene? calls
* b. Stay with the client with all medications are
1 point taken
c. Isolate the suicidal patient
A. The NA assists the patient to ambulate to the d. Search the client belongingness in his/her
bathroom and back to bed. presence for potentially harmful objects
B. The NA reminds the patient not to look at his feet
when he is walking. 45. Zoshima is being discharged after spending six
C. The NA performs the patient’s complete bath and days in the hospital, due to depression with suicidal
oral care. ideation. The psychiatric and mental health nurse
D. The NA sets up the patient’s tray and encourages knows that an important outcome has been met
patient to feed himself when the patient states:
*
SITUATION 1 point
Zoshima was rushed to the emergency room when
she was discovered to have taken over dosage of a. I can’t wait to get home and forget that this ever
sleeping pills. happened.”
b. “I have to leave here soon, if I want to make it to
41. Myths surround suicide but which of these the shelter before they run out of beds.”
should the nurse take as reality? c. “I have a list of support groups and a crisis line
* that I can call, if I feel suicidal.”
1 point d. “I feel so much better. If I continue to feel this
a. All suicide behavior should be taken seriously. It way, I can probably stop taking my medications
is a cry for help soon.”
b. Only psychotic persons try to kill themselves SITUATION
c. The suicide risk is over when improvement In the Psychiatric Ward, you are assigned to
follows a suicide crisis administer the medications for all the patients
d. Suicide attempts are manipulative plays admitted. Proper knowledge of the medications
42. Zoshima is on antidepressant treatment. A and their side effects are essential.
selective serotonin reuptake inhibitor was 46. Which information is most important for the
prescribed. This medication targets which part of nurse to include in a teaching plan for a male
the brain? schizophrenic client taking clozapine (Clozaril)?
* *
1 point 1 point
a. Putamen a. Monthly blood tests are necessary and will be
b. Frontal cortex conducted.
c. Basal ganglia b. Stop the medication gradually once you feel the
d. Hippocampus symptoms start subsiding.
43. The primary nursing diagnosis of Zoshima is: c. Your blood pressure must be monitored for
* hypertension.
1 point d. Report to your physician once sore throat or
fever occurs.
a. Risk for suicide
b. spiritual distress 47. What is non-antipsychotic medication used to
c. Loss of self-control treat some clients with schizoaffective disorder?
d. Ineffective coping *
1 point
44. All of these are basic suicide precautions
except: a. Lithium carbonate (Lithane)
* b. Imipramine (Tofranil)
1 point c. Chlordiazepoxide (Librium)
d. Phenelzine (Nardil)
48. Dana, a psychiatric client, is to be discharged 52. Elmma Rosario was to inject Vitamin B
with orders for haloperidol (haldol) therapy. When intramuscularly to another elderly patient. Before
developing a teaching plan for discharge, the nurse injecting, the nurse explained that the client may
should include cautioning the client against: feel some discomfort. This is an example of:
* *
1 point 1 point
a. Driving at night a. reducing pain receptor
b. Ingesting wines and cheese b. self-preservation
c. Staying in the sun c. anticipatory response
d. Taking medications containing aspirin d. distraction
49. Fred was newly diagnosed with anxiety 53. Mr. Colet, 68 years old, has a history of chronic
disorder. The physician prescribed buspirone back pain. He thinks that his family perceives him
(BuSpar). The nurse is aware that the teaching as a “weakling” because he often asks for pain
instructions for newly prescribed buspirone medication. Which of the following is the most
should include which of the following? therapeutic response of the nurse
* *
1 point 1 point
a. A warning about the incidence of neuroleptic a. “It seems that you are worried. Which matters to
malignant syndrome (NMS). you more? What people will say or getting relief
b. A reminder of the need to schedule blood work from your pain?”
in 1 week to check blood levels of the drug. b. “Taking pain medication as prescribed will help
c. A warning about the drugs delayed therapeutic you become more active.”
effect, which is from 14 to 30 days. c. “Chronic back pain is very difficult to manage, use
d. A warning that immediate sedation can occur pain medication because that is what it is for.”
with a resultant drop in pulse. d. “Don’t you think your family wants you to be
comfortable, and the only way is to take your
50. Which of these medications would cause the medicine?”
least dependence and side effects in a bipolar
patient? 54. Mang Sinok has chronic pain due to
* osteoarthritis but has impaired speech. Which of
1 point the following is the most appropriate to determine
his medication needs for pain?
a. Benzodiazepine *
b. MAOI 1 point
c. TCAs
d. SSRIs a. Observe typical pain behavior through facial
expressions
SITUATION b. Asking the client to rate his pain on a scale of 0 to
Nurse Elmma Rosario is assigned in the medical- 10 by writing on a magic slate
surgical unit and most of the clients assigned to her c. Medicate the client with analgesics as often as
were elderly clients. ordered
51. For a client complaining of mild d. Record frequency of patient’s complaint of pain
musculoskeletal pain, the nurse will anticipate that and administer medication accordingly
the treatment for this client’s level of discomfort 55. Aling Powks, 65, diabetic, complained of
will include which of the following? elevated blood glucose since she strained her back
* a week ago despite following her diet and drug
1 point prescription. Your best explanation would be:
a. Diazepam *
b. Meperidine hydrochloride 1 point
c. Acetaminophen a. Physiologic and psychologic stress can elevate
d. Fentanyl - Narcotic blood glucose level
b. Client is consuming more food as a coping patients. If you are a member of this team, which of
mechanism the following measures will you consider as the
c. It is usual occurrence among the elderly MOST appropriate to be implemented in
d. Parasympathetic stimulation from the body’s collaboration with the respiratory therapist?
normal response to pain *
1 point
SITUATION
The declining of patient satisfaction related to a. Perform regularly assessment of the client’s
nursing service per survey results as well as readiness to be extubated.
increased incidences of hospital acquired infection b. Consider orotracheal as preferred route of
during the past 6 months caused the nursing endotrachealintubalation.
service division to push the nursing units to c. Maintain head elevation at 30-45 degrees.
explore quality improvement projects. d. Suction endotracheal tube as prescribed in the
manual of procedures.
56. The intensive care unit (ICU) quality
improvement team decided to gather data to 59. Noise level in the ICU has always been a
determine probable causes of central line infection complaint in the patient satisfaction survey. Which
among the ICU patients. If you were the member of of the following tools can be recommended to the
the quality improvement team, which of the quality improvement team as most appropriate to
following data will you consider as MOST determine level of noise in the ICU.
appropriate to yield the most probable cause of *
central line infection? 1 point
*
1 point a. Questionnaire with clients and patients as
respondents
a. Nurses’ notes on hourly assessment of sites of b. Observation checklist
central line. c. Measurement device
b. Performed central line care interventions as d. Interview schedule form with nurses, clients and
observed. relatives as interviewees.
c. Daily every shift report of central line care
measures from bedside nurses. 60. During a group discussion, probable factors
d. Incidence of central line infection as reported by responsible for urinary tract infection incidences
infection control nurse. among the hospitalization clients in the medical
unit were being explored. Which of the following
57. The highest incidence of fall among the will you consider as the group of data which would
hospitalization patients is in the medical unit. The be LEAST helpful?
medical unit’s quality improvement team has
identified the probable causes of the incidences of 1. Diameter and length of Foley catheter
fall among their hospitalization patients. With the
data analyzed and findings organized, which of the 2. Length of time Foley catheter has been kept
following should the quality improvement team do indwelling
FIRST?
*
3. Age and sex of client
1 point
a. Implement fall prevention measures identified to 4. Daily physical activities of the client
be effective
b. Propose a list of nursing actions intended to 1. Relevant data regarding need for continuing
identify fall risks and preventive measures. indwelling catheter
c. Do a pilot study of the fall prevention measures *
to a small group of patients. 1 point
d. Brainstorm for a plan for an appropriate action
for change. a. 1,2,4
b. 3,4,5
58. Another group of quality improvement team in c. 1,2,3
the ICU conducted a project on ventilator d. 2,3,4
associated pneumonia incidences among ICU
SITUATION 65. To prevent deformities of Mrs. Daccu, the nurse
Mrs. Daccu, 65 years old, had an acute attack of includes in the nursing care plan:
pain, soreness and swelling on both knees. She is *
diagnosed with rheumatoid arthritis. 1 point
61. Nurse Kennethlyn is assessing the client. Which A. messaging the join with oil liniment
of the following is MOST likely to be assessed? B. Implementation of strictly prescribed diet
* C. Performing isometric exercises twice a day
1 point D. Alternate rest periods with active exercises
A. Early morning stiffness 66. The nurse notices that the comatose client
B. Nodules along the knees starts to lighten. She is aware that without
C. Joint for deformities protection, the client could fall or be injured. Which
D. Limited motions of joint of the following is the LEAST intervention?
*
62. The client is in the acute phase of rheumatoid 1 point
arthritis. In addition to the prescribed medication,
the physician orders application of heat and cold to A. Restrain the client to prevent from falling
manage arthritis pain. Which of the following B. Give adequate support when turning or moving
statements indicate that the client lacks C. Keep the side rails up on the bed
understanding in the application of heat and cold? D. Protect client’s head.
*
1 point 67. Following hip replacement after 24 hours the
client asks for assistance onto the bedpan. She is
A. “Cold application is applied for 20 min, then 20 placed in an orthopaedic bed and to facilitate the
minutes off” use of the bedpan, how should the nurse assist the
B. B.” Hot water bag should be covered with flannel client?
to prevent burns.” *
C. “Heat and cold can be applied as needed.” 1 point
D. “Heat producing liniments can be used while
applying heat and cold”. A. Pull on the trapeze to lift the pelvis extending
both legs
63. Nurse Kennethlyn is helping the client, who is B. Lifting the pelvis off the bed and turn gently
immobilized by pain, towards self-reliance and toward the operative side
independence. The nurse should approach the C. Assist the client in lifting the pelvis
problem with which of the following: D. Elevate the pelvis using the trapeze involving the
* unaffected upper extremity and unoperated leg
1 point
68. Caloy, an elderly client, is to be discharged after
A. Set a specific goal sustaining a sprain from fall while negotiating the
B. Set a positive attitude toward an eventful last step of the stairs. The daughter asks the nurse
outcome how to promote safety in the stairways and
C. Need for a member of the family during the pain hallways in the home. The nurse recommends extra
episode lighting at the stairways and suggests repainting
D. Recognize that little can be accomplished. the hallways with:
*
64. The nurse should know that a client with 1 point
rheumatoid arthritis will most often have pain and
limited movements of the joints: A. Red and yellow
* B. Blue and green
1 point C. Black and white
D. Cream and white
A. Resulting from non-adherence to prescribed diet
B. After excessive exercises 69. Filomena, 32 years old has problem with the
C. Because of inactivity upon awakening in the olfacatory nerve. They live in a thickly populated
morning area and is concerned for the safety of her 3 young
D. During cold weather
children. What measure should the nurse D. D.” I will avoid pizza, any food with cheese and
recommend for home safety? processed meat”.
*
1 point 73. While giving Chlorpromazine (Thorazine) to
client Mica,medication when she observes this side
A. Install additional lighting for visibility effect:
B. Participate in fire prevention training *
C. Mild water heater temperature 1 point
D. Install smoke detector device
A. Shuffling gait
70. Lola Barbie , 76 years old is living alone. Her B. Fine tremors
married daughter visits her from time to time. She C. Yellow sclerae
can do activities of daily living with limited D. Facial grimacing
assistance and seems to independently physically.
Which of the following measure should be 74. Another client in the ward, Lando, is given.
recommended to reduce sensory deprivation? Thorazine (Chlorpromazine). This medication has
* several side effects. Which side effect should cause
1 point nurse to be MOST concerned?
*
A. Encourage acquaintances to come to house for a 1 point
chat
B. Redecorate the house and provide a separate A. uncomfortable sun burns
room B. Sore throat, fever, decreased white blood cell
C. Provide pictures of family members count
D. Invite friends often to share meals at home C. Tremors, inability to stand still
D. Low blood pressure upon getting up from bed.
SITUATION
Many clients in psychiatric unit receive 75. Clients on antipsychotic medications usually
antipsychotic meds, also referred to as receive aniparkinson drugs to reduce Parkinson
Neuroleptics. like side effects. What medication would the nurse
expect the client to receive?
71. Clients may be shifted from typical to atypical *
antipsychotic medications because of its minimal 1 point
extrapyramidal side effects. A common
extrapyramidal symptom that is very unpleasant A. Congentin (Benztropine)
and intolerable to clients is called akathisia. This is: B. Nardil (Phenelzine)
* C. Fluphenazine (Prolixin)
1 point D. Flioxetine (Prozac)

A. Upward rolling of the eyes SITUATION


B. Inability to sit or stand still Stress of hospitalization can lead to difficulties
C. Pill rolling movement of hands between nurses & patients. Ff. Are situations that
D. Stiffening of client’s neck nurses presented during a monthly nursing circle.

72. Health instructions about Haldol (haloperidol) 76. Jury asked the nurse to have an “out on pass”
has been given to Albert while in hospital & before privilege for the weekend but his request was not
his discharge. Client correctly understood health granted by the nurse. He remarked, “I thought you
techniques of nurse when he says: really liked me”. A Therapeutic response of the
* nurse would be:
1 point *
1 point
A. “I will immediately report any episode if
diarrhea or vomiting to my doctor” A. Say, “I understand, you feel bad but of course, I
B. “I will drink about 2 liters of fluids daily and like you”.
expect to urinate frequently”. B. Say as a matter of fact, “Yourbehavior did not
C. “I will wear long sleeve clothing and sun block meet criteria for out on pass privilege.”
when i go out”. C. Ignore Jurry’s remark
D. Be transparent and express disapproval openly. Mrs. Centeno is an 85 year old woman who has
“You upset me with your remark.” been hospitalized due to a urinary tract infection
and dehydration. She has Alzheimer’s disease,
77. The dynamics of behavior underlying osteoporosis, and a tendency to wander. She has an
manipulative behavior explain that it is a behavior IV in her left forearm, which was difficult to
of: establish. Concerned that Mrs. Centeno might pull
* out her IV and wander off the floor, staff is
1 point considering possibility of using restraint on her.
A. A sense of security and control 81. The staff is considering the possibility of using
B. Exhibiting uncooperative and hostile behavior restraint on Mrs. Centeno, however, she repeatedly
C. Reducing patient’s anxiety declares that she does not want to be restrained.
D. Sensing fear of other people The staff is faced with an ethical dilemma of
78. Carlota, an elderly client idealizes some nurses autonomy versus:
as “terrific” , “the best”, or “so understanding”, but *
refers to others as “mean”, or” indifferent”. This 1 point
behavior can be understood by the staff as: A. Beneficence
* B. Veracity
1 point C. Fairness
A. Avoiding taking responsibility for her own D. Justice
behavior and underlying feelings 82. With a history of osteoporosis and a tendency
B. An understandable behavior for an elderly that to wander, which of the following should be a
must not be taken seriously priority?
C. An invitation to have social & intimate *
relationship w/ her nurse 1 point
D. Immature and childish behavior
A. Request for a sitter
79. A patient with delirium touches the nurse B. Wheelchair privilege commode
inappropriately. The therapeutic response of the C. Prevention of fall
nurse would be to: D. Provision of a bedside
*
1 point 83. Which of the following would be LEAST likely
appreciated by Mrs. Centeno?
A. Ask for patient’s name and if whether he is aware *
where he is. 1 point
B. Remove the patient’s hand while saying calmly,
“I’m the nurse and this is a hospital.” A. Playing a table board game
C. Say nothing and just go on with the usual nursing B. Singing to or with her
interventions C. Going through family picture album
D. Say her name,”I’m Cathy, I’m your nurse.” D. Listening to old familiar music
80. The staff nurses have differing emotional 84. The nurse aims at highest level of self-care.
reactions to the use of limit setting. Some staff Which of the following will the nurse minimize?
views it as unprofessionally punitive and uncaring. *
The MOST appropriate approach to address the 1 point
nursing concern is through.
* A. providing mouth swabs
1 point B. Using clothing w/ buttons and zippers
C. Hand and body lotion
A. Counseling with the nursing supervisor D. Labeling clothing items
B. Seminar-workshop
C. Nursing conference 85. Mrs. Centeno has a dietary privilege of food
D. Brainstorming session preferences. Which question is MOST effective to
communicate with her?
SITUATION *
1 point
A. Which way would you want your egg done? 90. You disclosed to your co-nurse that your
Scrambled? Sunny side up? With vegetable mix? Or patient is HIV positive. Which of the following
boiled egg? actions the patient may do? The patient:
B. Do you want fried egg or boiled egg? *
C. How would you want to have your egg done? 1 point
D. What is your favourite egg recipe?
A. May sue you for breach of confidentiality
SITUATION B. May forgive you
The concept of testimonial privilege applies only in C. Can file an administrative case again you for
a court-related proceeding. As a professional nurse, unethical behavior
you should have a clear understanding of this D. Can confront you
concept.
SITUATION
86. Communication between two people is NOT You are the school nurse of the third district of the
considered as testimonial privileges: Province of Rizal. You were invited by the
* Federation of the Parent-Teacher Association of
1 point the province to give a lecture on Seizure Disorder.
An open-forum was held after your lecture.
A. Between a teacher and a student
B. Between husband and wife 91. A parent whose daughter was newly diagnosed
C. Between a male and a female going steady with Epilepsy asked you what should be a part of
D. Between a lawyer and his client your teaching plan for her daughter who is being
discharged on a regimen of Dilantin. Which of the
87. In a testimonial privilege, the right to reveal following would be your correct answer?
privileged information belongs to the *
* 1 point
1 point
A. Reporting signs of infection
A. Clergy B. Drinking plenty of fluids
B. The listener C. Brushing teeth after each meal
C. lawyer D. Having someone with the child during waking
D. the person who spoke hours
88. In which of the following situations does 92. You asked some school teachers to do a role
testimonial privilege between health professionals play on the management of seizure in the
and patients exist? classroom. Which of the following actions when
* PERFORMED First indicates that your lecture has
1 point been successful?
A. If it is established by the code of ethics of the *
health professional 1 point
B. If it is established by law A. Moving the child to the principal's office for
C. If it is within the social norms privacy
D. If both parties agreed to it B. Placing a padded tongue blade between the
89. In which of the following situations a privileged child's teeth
communications between a nurse and a patient C. Removing any sharp objects that can harm the
may be breached? child
* D. Asking the other children what happened before
1 point the seizure

A. If the information will cause harm to the nurse 93. A parent asked you what they should do to
B. It should never be breached promote the growth and development of their 7-
C. If there is a threat in the health of the general year old son who has just been diagnosed with a
population seizure disorder. You should instruct the parents
D. If the information will cause harm to the patient that:
*
1 point
A. there is a potential for a learning disability and D. "Nursing practice is based on research findings"
their son may need tutoring to achieve his grade
level. 97. Research utilization is an important aspect of
B. The child will likely have normal intelligence and safe, quality care provided by the nurse to her
be able to attend regular school. patients. Research utilization refers to:
C. There will be problems associated with social *
stigma and home schooling must be considered. 1 point
D. The child will need activity limitation and will A. the development of nursing knowledge to
not be able to perform as well as his peers. improve clinical practice
94. A mother with a daughter who has an B. the application of research findings into clinical
occasional generalized seizure wants her daughter practice
to join the summer camp on their church. The C. the integration of the best research evidence,
mother asked for your advice on planning for the clinical expertise and patient preferences in
camping experience. Which of the following providing safe, quality patient care
activities should be avoided? D. the application of tested theories in the care of
* the patients
1 point 98. Which of the following research methods has
A. rock climbing for its purpose to describe social processes present
B. tennis within human interactions?
C. hiking *
D. swimming 1 point

95. You discussed measures, other than A. Phenomenology


medication, to lower temperature of children with B. Participatory action research
febrile seizure. Which of the following statements C. Grounded theory
indicted that the participants understand the D. Case study
topic? 99. In a clinical question, "Is breastfeeding more
* effective in increasing the birth weight of preterm
1 point infant than adding corn oil to the infant formula?"
A. "We'll make the bath water cold enough to make what is the intervention of interest?
him shiver" *
B. "we'll add extra blankets when he says he is cold" 1 point
C. "We'll use a solution of half alcohol and half A. Breastfeeding
water when sponging him" B. Preterm infant
D. "We'll wrap him in a blanket if he starts C. Increasing the birth weight
shivering" D. Adding corn oil to the infant formula
SITUATION 100. In a research question, "What is the
You are a staff nurse in the Orthopedic Unit of the relationship between wound healing and nutrition
Department of Surgery of the Hospital. among elderly patient with hip surgery?" which
96. You are assisting in a research study on one is the dependent variable?
assessing patient's reactions to the use of new *
dressing material. A medical student questions the 1 point
credibility of the nursing research. Your response A. Elderly patient
would be: B. hip surgery
* C. Wound healing
1 point D. Nutrition
A. To keep quiet
B. "Nursing research is essential for the
development of nursing science"
C. "Doing research is one of the competencies of
professional nurses"

You might also like